Тест по информатике «Системы счисления»

Просмотр содержимого документа
«Тест по информатике «Системы счисления»»

Тест

по информатике

“ Система счисления”

Версия от 25.11.2014 г. Последнюю версию конструктора смотрите на сайте «Тестирование в MS PowerPoint» http://www.rosinka.vrn.ru/pp/

Введите фамилию и имя

Начать тестирование

10

15

мин.

Всего заданий

Время тестирования

Совокупность знаков, с помощью которых записываются числа, называется:

системой счисления

1

цифрами системы счисления

2

алфавитом системы счисления

3

основанием системы счисления

4

Далее

1

Задание

1 бал.

Чему равен результат сложения двух чисел, записанных в римской системе счисления: MCM + LXVIII?

1168

1

1968

2

2168

3

1153

4

Далее

2

Задание

1 бал.

Число 3001011 может существовать в системах счисления с основаниями:

2

1

3

2

4

3

8

4

10

5

Далее

Выберите все правильные ответы!

3

Задание

1 бал.

Двоичное число 100110 в десятичной системе счисления записывается так:

36

1

38

2

37

3

46

4

Далее

4

Задание

1 бал.

Установите соответствие между римскими и арабскими числами:

49

15

90

51

XV

XC

XLIX

LI

Далее

Укажите все правильные позиции!

5

Задание

1 бал.

Системы счисления бывают:

Непозиционные

1

Сложнопозиционные

2

Позиционные

3

Беспозиционные

4

Далее

Выберите все правильные ответы!

6

Задание

1 бал.

В восьмеричной системе счисления присутствуют такие символы:

0,1,2,3,4,5,6,7,8

1

1,2,3,4,5,6,7,8

2

0,1,2,3,4,5,6,7

3

1,2,3,4,,5,6,7

4

Далее

7

Задание

1 бал.

Самая древняя система счисления:

двоичная

1

единичная

2

десятичная

3

Далее

8

Задание

1 бал.

ЭВМ выполняет арифметические расчеты в системе счисления:

десятичная

1

двоичная

2

шестнадцатеричная

3

восьмеричная

4

Далее

9

Задание

1 бал.

Установите соответствие между числами в 16-ричной и 10-ричной системах счисления:

E

F

C

D

A

13

14

12

15

10

Итоги

Укажите все правильные позиции!

10

Задание

1 бал.

Результаты тестирования

Подождите!

Идет обработка данных

Оценка

Правильных ответов

Набранных баллов

В конструкторе использована идея перемещения объектов в режиме просмотра демонстрации, предложенная Гансом Хофманом (Hans Werner Hofmann [email protected])

Ошибки в выборе ответов на задания:

Снова

Выход

Всего заданий

бал.

Затрачено времени

10

Итоговый тест по информатике в 8 классе

Итоговый тест по информатике 8 класс

  1. Совокупность знаков, с помощью которых записываются числа, называется:

  1. Система счисления

  2. Алфавит системы счисления

  3. Основание системы счисления

  1. Чему равен результат сложения двух чисел, записанных римскими цифрами: MCM+LXVIII?

  1. 1168

  2. 1968

  3. 2168

  4. 1153

  1. Чему равно двоичное число число 100110 в десятичной системе счисления?

  1. 36

  2. 38

  3. 37

  4. 46

  1. Чему равен результат сложения чисел 1102 и 128?

  1. 610

  2. 1010

  3. 100002

  4. 178

  1. На перекрестке произошло дорожно-транспортное происшествие, в котором в котором участвовали автобус (А), грузовик (Г), легковой автомобиль (Л) и маршрутное такси (М). свидетели произошедшего дали следующие показания. Первый свидетель считал, что первым на перекресток выехал автобус, а маршрутное такси было вторым. Другой свидетель полагал, что последним на перекресток выехал легковой автомобиль , а вторым был грузовик. Третий свидетель уверял, что автобус выехал на перекресток вторым, а следом за ним – легковой автомобиль. В результате оказалось, что каждый из свидетелей бал прав только в одном из своих утверждений. В каком порядке выехали машины на перекресток? В вариантах ответов перечислены подряд без пробелов первые буквы названий транспортных средств в порядке их выезда на перекресток:

  1. АМЛГ

  2. АГЛМ

  3. ГЛМА

  4. МЛГА

  1. Какое из логических выражений соответствует следующей схеме?

  1. A&B

  2. AvB

  3. A&B

  4. A&B

7. Алгоритм – это:

  1. правила выполнения определенных действий;

  2. набор команд для компьютера;

  3. протокол для вычислительной сети;

  4. описание последовательности действий, строгое исполнение которых приводит к решению поставленной задачи за конечное число шагов.

8. Свойство алгоритма, заключающееся в отсутствии ошибок, алгоритм должен приводить к правильному результату для всех допустимых входных значений, называется:

  1. результативность;

  2. массовость;

  3. дискретность;

  4. конечность.

9. Свойство алгоритма, заключающееся в том, что один и тот же алгоритм можно использовать с различными исходными данными, называется:

  1. результативность;

  2. массовость;

  3. конечность;

  4. детерминированность.

10. К какому виду алгоритмов модно отнести алгоритм, схема которого представлена ниже?

  1. Линейный

  2. Разветвляющийся

  3. Циклический

  4. С параметром

11. К какому виду алгоритмов модно отнести алгоритм, схема которого представлена ниже?

  1. Линейный

  2. Разветвляющийся

  3. Циклический

  4. С параметром

12. К какому виду алгоритмов модно отнести алгоритм, схема которого представлена ниже?

  1. Линейный

  2. Разветвляющийся

  3. Циклический

  4. С параметром

13. К какому виду алгоритмов модно отнести алгоритм, схема которого представлена ниже?

  1. Линейный

  2. Разветвляющийся

  3. Циклический

  4. С параметром

14. выберите целочисленный тип данных в программе Паскаль?

  1. Real

  2. Integer

  3. Boolean

  4. String

15. Напишите операторы ввода вывода данных на языке Паскаль?

16. напишите структуру программы Паскаль?

Ответы к тесту

8 класс

  1. b

  2. b

  3. b

  4. c

  5. b

  6. d

  7. d

  8. a

  9. b

  10. a

  11. b

  12. c

  13. d

  14. b

Контрольный тест по информатике на тему «математические основы информатики». Математические основы информатики Итоговый тест по теме математические основы информатики

Тест по информатике Математические основы информатики предназначен для учащихся 8 класса. Тест содержит 20 вопросов. В конце теста имеются ответы.

1. Совокупность знаков, при помощи которых записываются числа, называется:

а) системой счисления
б) цифрами системы счисления
в) алфавитом системы счисления
г) основанием системы счисления

2. Чему равен результат сложения двух чисел, записанных римскими цифрами: МСМ + LXVIII?

а) 1168
б) 1968
в) 2168
г) 1153

3. Число 301011 может существовать в системах счисления с основаниями:

а) 2 и 10
б) 4 и З
в) 4 и 8
г) 2 и 4

4. Двоичное число 100110 в десятичной системе счисления записывается как:

а) 36
б) 38
в) 37
г) 46

5. В классе 110010 2 % девочек и 1010 2 мальчиков. Сколько учеников в классе?

а) 10
б) 20
в) 30
г) 40

6. Сколько цифр 1 в двоичном представлении десятичного числа 15?

а) 1
б) 2
в) 3
г) 4

7. Чему равен результат сложения чисел 110 2 и 12 8 ?

а) 6 10
б) 10 10
в) 10000 2
г) 17 8

8. Ячейка памяти компьютера состоит из однородных элементов, называемых:

а) кодами
б) разрядами
в) цифрами
г) коэффициентами

9. Количество разрядов, занимаемых двухбайтовым числом, равно:

а) 8
б) 16
в) 32
г) 64

10. В знаковый разряд ячейки для отрицательных чисел заносится:

а) +
б) —
в) 0
г) 1

11. Вещественные числа представляются в компьютере в:

а) естественной форме
б) развёрнутой форме
в) нормальной форме с нормализованной мантиссой
г) виде обыкновенной дроби

12. Какое предложение не является высказыванием?

а) Никакая причина не извиняет невежливость
б) Обязательно стань отличником
в) Рукописи не горят
г) 1011 2 = 1 х 2 3 + 0 х 2 2 + 1 х 2 1 + 1 х 2 0

13. Какое высказывание является ложным?

а) Знаком v обозначается логическая операция ИЛИ
б) Логическую операцию ИЛИ иначе называют логическим сложением
в) Дизъюнкцию иначе называют логическим сложением
г) Знаком v обозначается логическая операция конъюнкция

14. Для какого из указанных значений числа X истинно высказывание
((X ?

а) 1
б) 2
в) 3
г) 4

15. Для какого символьного выражения верно высказывание:
«НЕ (Первая буква согласная) И НЕ (Вторая буква гласная)» ?

a) abcde
б) bcade
в) babas
г) cabab

16. Некоторый сегмент сети Интернет состоит из 1000 сайтов. Поисковый сервер в автоматическом режиме составил таблицу ключевых слов для сайтов этого сегмента. Вот её фрагмент:
сканер — 200
принтер — 250
монитор — 450

Сколько сайтов будет найдено по запросу принтер | сканер | монитор , если по запросу принтер | сканер было найдено 450 сайтов, по запросу принтер & монитор — 40, а по запросу сканер & монитор — 50?

а) 900
6) 540
в) 460
г) 810

17. Какому логическому выражению соответствует следующая таблица истинности?

A B F
0 0 1
0 1 1
1 0 1
1 1 0

18. Когда сломался компьютер, его хозяин сказал: «Оперативная память не могла выйти из строя». Сын хозяина компьютера предположил, что сгорел процессор, а жёсткий диск исправен. Пришедший специалист по обслуживанию сказал, что, скорее всего, с процессором всё в порядке, а оперативная память неисправна. В результате оказалось, что двое из них сказали всё верно, а третий — всё неверно. Что же сломалось?

а) оперативная память
б) процессор
в) винчестер
г) процессор и оперативная память

19. На перекрёстке произошло дорожно-транспортное происшествие, в котором участвовали автобус (А), грузовик (Г), легковой автомобиль (Л) и маршрутное такси (М). Свидетели происшествия дали следующие показания. Первый свидетель считал, что первым на перекрёсток выехал автобус, а маршрутное такси было вторым. Другой свидетель полагал, что последним на перекрёсток выехал легковой автомобиль, а вторым был грузовик. Третий свидетель уверял, что автобус выехал на перекрёсток вторым, а следом за ним — легковой автомобиль. В результате оказалось, что каждый из свидетелей был прав только в одном из своих утверждений. В каком порядке выехали машины на перекрёсток? В вариантах ответов перечислены подряд без пробелов первые буквы названий транспортных средств в порядке их выезда на перекрёсток.

Учебный план

№_ГАЗЕТЫ

Лекция

Лекция 1. Что такое “математические основы информатики”. Почему информатику нередко считают близкой род-
ственницей математики? Верно ли это? Возможна ли информатика без математики? Какая математика нужна для освоения
информатики? Может ли школьная математика дать основу для информатики?

Информация и ее кодирование. Математика кодов. Коды, исправляющие ошибки. Экономное кодирование.

Лекция 2 . Математические модели формальных исполнителей. Что такое формальная обработка информации? Конеч-
ные автоматы. Что первично: язык или исполнитель? Грамматика языка. Распознаваемые языки. Универсальные исполни-
тели (машина Тьюринга, машина Поста).
Лекция 3 . Алгоритм и его свойства. Алгоритмическая неразрешимость. Вычислимость. Сложность.
Контрольная работа № 1.
Лекция 4. Графы . Графы и орграфы. В каких задачах они возникают? Различные свойства графов (эйлеровость, гамильто-
новость, планарность, двудольность). Сети. Потоки в сетях. Представление графов. Основные алгоритмы на графах.
Лекция 5 . Логические модели в информатике. Алгебра высказываний. Булевы функции. Нормальные формы. Полные
классы булевых функций. Релейно-контактные схемы. Вентили. Математические модели процессора и памяти компьютера. Предикаты и отношения. Реляционная алгебра. Теоретические основы реляционных СУБД. Языки логического программирования и их математическое основание.
Контрольная работа № 2.
Лекция 6 . Компьютерная теория чисел и вычислительная геометрия. Зачем нужна теория чисел в компьютерных
науках? Гонка за простыми числами. Как разложить число на множители? Чем отличается теоретическая геометрия от
вычислительной? Почему гладко на бумаге, но коряво на компьютере? Основные правила и алгоритмы вычислительной
геометрии.
23/2007Лекция 7 . Защита информации . Защита символьной информации. Что нужно защищать? Электронная подпись. Системы
верификации. Криптосистемы с открытым ключом. Защита графической информации. Математика электронных водяных знаков.
24/2007Лекция 8 . Основы методики преподавания математических основ информатики.
Итоговая работа

Теория чисел и геометрия… Два раздела математики, которые издревле придавали ей дух игры высокого интеллекта, в которой царствовала красота логических построений. На, казалось бы, невинный и естественный вопрос ученика “Какая польза от геометрии?” Евклид велел рабу дать ученику монету и прогнать его. “Он ищет пользу в геометрии!” — с возмущением ответствовал Евклид.

Бесполезность для народного хозяйства теории чисел представляется еще более очевидной. Ну кому, скажите, стало легче жить после того, как был получен ответ на вопрос, имеет ли решение в натуральных числах уравнение x n + y n = z n , если n > 2? А ведь этот вопрос, известный как Великая проблема Ферма, волновал математиков более 300 лет. И даже доказанный еще Евклидом факт, что простых чисел бесконечно много, вряд ли способен помочь увеличению национального валового продукта.

Другое дело компьютер. Вещь, без сомнения, полезная. Правда, ограниченная. И количество ячеек памяти, хоть и большое, но конечное. И разрядность каждой ячейки тоже конечна. Так что числа типа или p в силу своей иррациональности в компьютер “не помещаются”. А как увидеть на экране компьютера бесконечную прямую? А если прямых две, и на глаз они выглядят как параллельные? А вдруг точка пересечения все-таки есть, но где-то далеко-далеко?

Конечное и бесконечное, рациональное и иррациональное — вот противоположности, представленные в математике ярко, можно сказать, в обнаженном виде. Иррациональное и бесконечное завораживает, как взгляд в бездонные глубины космоса или в основы мироздания. Это то, что недоступно ощущению, а подвластно только разуму. И маленький ограниченный компьютер, каким бы мощным он ни был, кажется здесь бесполезным.

В 1742 г. Гольдбах в письме к Эйлеру высказал гипотезу, что каждое нечетное число, начиная с 7, представимо в виде суммы трех простых чисел. Почти 200 лет решение проблемы не удавалось сдвинуть с мертвой точки. В 1934 г. академик И.М. Виноградов доказал, что существует некоторое число N , такое, что все нечетные числа, большие чем N , представимы в виде суммы трех простых чисел. Причем это число N было вычислено (К.Бороздкин): N = ее 16,038. Оно имеет всего-навсего 4 008 659 цифр. И никакой бесконечности. Достаточно проверить утверждение для всех нечетных чисел, меньших N , и проблема Гольдбаха будет решена. Человеку такое, конечно, не под силу. Но, к сожалению, и компьютерным системам пока эта задача не по зубам.

Во второй половине XX века накопилось немало теорем теории чисел, которые утверждают, что все хорошо, начиная с некоторого N , т.е. там, далеко, где нас нет. И дотянуться до этой границы простым применением компьютера мы, увы, не можем. Получилось так, что теория чисел стала поставщиком задач, экстремальных для компьютера по объемам памяти, по быстродействию, по эффективности разрабатываемых алгоритмов. Кроме того, для математиков необходимо доказательство, что созданная компьютерная программа работает правильно. А ведь вручную результаты не перепроверишь. Потребовалась разработка весьма тонких методов доказательства правильности алгоритмов и программ (заметим, что это, вообще говоря, не одно и то же). О некоторых таких методах мы рассказывали в лекции 3. Здесь же мы расскажем, как математики помогают обуздать бесконечность и иррациональность.

Можно сказать, что эта лекция о содружестве чистой математики и компьютерной науки.

§ 24. Использование компьютера в теоретико-числовых исследованиях

Приведенные выше общие рассуждения о применении компьютерных вычислений в доказательстве математических утверждений, наверно, достаточно прозрачны. Тем не менее представляется полезным увидеть, как реально они воплощаются. Мы, конечно, не возьмемся решать ту или иную знаменитую теоретико-числовую проблему, а продемонстрируем это на не очень сложной задаче.

Задача 1. Верно ли, что для любой цифры N, отличной от нуля, существует натуральное число, оканчивающееся на цифру N и такое, что ее перенос в начало числа приводит к увеличению числа в N раз?

При N = 1 положительный ответ очевиден: годится любое число, составленное из более чем одной 1, например, 11. А как обстоит дело с другими значениями N ?

Ненулевых цифр совсем немного, можно попытаться получить ответ для каждой из них. Начнем с N = 2. Число оканчивается на 2, и после переноса этой цифры в начало должно получиться в 2 раза большее число, и потому оно оканчивается на 4. Значит, исходное число оканчивается на 42. Тогда после переноса цифры 2 в начало должно получиться число, оканчивающееся на 84. Значит, исходное число оканчивается на 842. Продолжая рассуждение, получаем, что предшествующая цифра 6. Процесс пошел. Но где гарантия, что он закончится?

Если у читателя хватит терпения, то он убедится, что в данном случае его ждет счастливое завершение — на семнадцатом шаге получится число 105 263 157 894 736 842, удовлетворяющее требованиям задачи.

Описанный выше процесс нетрудно запрограммировать, но сколько времени ждать ответ? А вдруг для некоторого N такого числа не существует? Тогда программа будет работать вечно 1 .

И снова на помощь приходится призвать математику. Во-первых, изменим немного условие задачи, предложив рассматривать более общую ситуацию 2 .

Задача 2. Известно, что число увеличивается в K раз от перестановки последней цифры в начало. При каких K это может происходить? Для каждого такого K найти наименьшее число, удовлетворяющее данному условию.

В этой задаче в отличие от задачи 1 не требуется, чтобы само число тоже начиналось на K ; более того, натуральное число K не обязано быть цифрой.

Решение. Обозначим через A исходное число, через B — число, получающееся из A перестановкой последней цифры в начало. Тогда K = B /A . Случай K = 1 интереса не представляет, поскольку тогда A = B и наименьшее число, как мы отмечали, равно 11. (Заметим, между прочим, что и в этом случае переставляемая цифра совпадает с K .) Поэтому в дальнейшем будем считать, что K 2.

Прежде всего разберемся, сколькизначным может быть K . Поскольку числа B и A имеют одно и то же количество цифр, отношение B к A не превосходит 9. Так что K 9 (т.е. можно сказать, что само K всегда является “цифрой”).

Пусть X — последняя цифра числа A (она же первая цифра числа B ), а Y — число, образованное всеми цифрами числа A , кроме последней. Пусть также n — количество цифр в числе Y . Тогда A = 10 · Y + X , а B = X · 10 n + Y . Тем самым, получаем равенство:

X · 10 n + Y = K (10 · Y + X ),

.

Знаменатель 10K – 1 всегда двузначен. Вот какие значения он принимает:

В третьей строке этой таблицы приведено в виде несократимой дроби значение множителя перед X при n = 1. Это означает, что ни для какой цифры X при n = 1 значение Y не получается целым. Значит, n 2.

Тот факт, что Y имеет в записи n цифр, означает, что 10 n –1 Y n . Отсюда для X получаем двойное неравенство:

.

Левая часть этого неравенства легко преобразуется к виду:

Поскольку n 2 и K 9, дробь (10n –1 – K ) : (10n K ) положительна и меньше 1. Учитывая, что X – целое число, левую часть неравенства можно записать K X .

Займемся теперь правой частью этого неравенства. Аналогичные преобразования показывают, что она равна

.

Поскольку K 9 и n 2, дробь 9K / (10 n K ) положительна и меньше 1. Учитывая, что X — целое число, правую часть неравенства можно записать X 10K – 1. Впрочем, это неравенство нам мало что дает, поскольку по условию X и так меньше 9.

Итак, мы выяснили, что перемещаемая цифра не может быть меньше, чем K . Так что в задаче 1 просто взято наименьшее возможное значение для перемещаемой цифры.

Теперь займемся главным вопросом задачи 2: при каких же K найдется такое n , что число Y окажется целым? Иными словами, для каких K существует n , при котором 10 n K делится на 10K – 1? Здесь мы воспользуемся следующей идеей. Для удобства дальнейших рассуждений обозначим число 10K – 1 буквой m . Ясно, что число m не имеет с числом 10 общих делителей, отличных от 1. В частности, никакая степень числа 10 не делится на m без остатка.

Рассмотрим последовательность степеней числа 10:

1 = 10 0 ; 10 1 ; 10 2 ; 10 3 ; …; 10 m-1 ,

и последовательность остатков от деления этих чисел на m . Все эти остатки, как было сказано, отличны от нуля. Однако различных ненулевых остатков при делении на m всего лишь m – 1. Значит, в данной последовательности степеней найдутся два числа с одинаковыми остатками при делении на m . Пусть это 10 a и 10 b , где
0 a b m – 1. Тогда 10 b – 10 a делится на m . Но 10 b – 10 a = 10 a (10 b-a – 1), а число m не имеет с числом 10 неединичных общих делителей. Поэтому на m в этом произведении делится число 10 b-a – 1. Обозначим b a через t .

Таким образом, нами доказано, что найдется такой положительный показатель степени t m – 1, что 10 t – 1 делится на m . Мы можем считать, что t выбрано наименьшим с тем свойством, что 10 t – 1 делится на m . Иными словами, среди степеней 10 1 ; 10 2 ; 10 3 ; …; 10 t-1 ни одна не дает остаток 1 при делении на m . А начиная с 10t остатки будут повторяться в той же последовательности. Тем самым, никаких других остатков при делении на m степеней числа 10, кроме тех, которые встречаются в ряду 1 = 10 0 ; 10 1 ; 10 2 ; 10 3 ; …; 10 t-1 , нет. Чтобы на m делилось число 10 n K , нужно K иметь остатком при делении подходящей степени числа 10 на m . А выяснить это можно, составив несложный циклический алгоритм, поскольку известно, какое наибольшее число степеней придется рассматривать, чтобы понять, встретится ли нужный остаток: не более чем m – 1. Вот соответствующий алгоритм:

алг задача_2 (арг цел K)

дано 2 K 9

надо | напечатать показатель степени числа 10,

| дающей в остатке K при

| делении этой степени на 10K–1.

нач цел X, N

нц пока не (X = K или N = M)

X:= mod(10*X, M)

то вывод N

иначе вывод «Такой степени нет»

Результаты работы этого алгоритма представлены в следующей таблице:

Поскольку ни при каком допустимом значении K алгоритм не выдал сообщение об отсутствии подходящей степени, мы доказали следующую теорему:

Теорема. Для каждого натурального K 9 существует число, которое увеличивается в K раз от перестановки последней цифры в начало.

Обратите внимание, как при доказательстве данной теоремы применялся компьютер. Во-первых, сначала была доказана конечность числа вариантов, подлежащих перебору, во-вторых, с помощью составленного алгоритма мы искали не само число, а проверяли выполнение некоторого условия, необходимого и достаточного для существования числа с нужным свойством.

Впрочем, пользуясь полученной таблицей, легко указать и сами числа с требуемым свойством — в качестве X надо взять K и воспользоваться полученной формулой для Y , чтобы найти требуемое там число A :

Желающие могут получить теперь и запись числа A цифрами (например, при K = 5 число А = 510 204 081 632 653 061 224 489 795 918 367 346 938 775; а самое длинное число получается при K = 6 — оно содержит 58 цифр).

Заметим, что не все приведенные ответы к задаче 1 являются ответами к нашей задаче 2: при K = 5 (и только при этом значении K ) 3 найдется меньшее число, удовлетворяющее условию задачи. Это число получается, если X взять равным 7. Тогда нужно будет разыскать степень числа 10, дающую в остатке 5 при делении не на 49, а всего лишь на 7. Показатель такой степени равен 5. Искомое число в этом случае 10(10 5 – 5) / 7 + 7 = 142 857. Это число просто “малютка” в сравнении с приведенным выше 42-значным числом, которое получается как ответ к задаче 1.

Конечно, это всего лишь демонстрация того, как компьютер может применяться к доказательству теоремы. На самом деле если поразмышлять еще немного, то можно доказать следующее утверждение: если 10 t – 1 делится на 10K – 1, то 10 t-1 при делении на 10K – 1 обязательно дает остаток K . Тем самым, в доказательстве теоремы можно было обойтись и без компьютера. Однако на сегодняшний день существует целый ряд математических утверждений, доказательство которых без компьютера пока получить не удается. Вот только один пример. Давно известно (с XVII века) и нетрудно доказать средствами школьной алгебры, что число 2 n – 1 может быть простым лишь при условии, что само число n является простым. Числа вида 2 n – 1 получили даже специальное название — числа Мерсенна — по имени средневекового монаха, обратившего внимание математиков на эти числа 4 . К сожалению, обратное утверждение неверно: не каждое число вида 2 n – 1 при простом n является простым. Например, число 2 11 – 1 = 2047 = 23 · 89 является составным. И хотя неизвестно, бесконечно ли много простых чисел Мерсенна, они и сегодня служат одним из источников больших простых чисел. До изобретения компьютеров самым большим известным простым числом Мерсенна было число 2 127 – 1, имеющее 39 цифр. И это было вообще самое большое известное в то время простое число. Появление компьютеров сразу увеличило наше знание простых чисел Мерсенна. Уже в 1952 году были обнаружены сразу 5 простых чисел: 2 521 – 1; 2 607 – 1; 2 1279 – 1; 2 2203 – 1; 2 2281 – 1. Последнее из них имеет 687 цифр; даже для современных компьютеров разложение такого числа на множители — дело весьма напряженное. Выход из этой сложной ситуации состоит в том, что для доказательства простоты числа Мерсенна используется так называемый “критерий Люка”. Он заключается в следующем. Для выбранного простого числа р рекуррентно строится последовательность s 1 , s 2 , s 3 , …, s n , где s 1 = 4, а s k+1 — остаток при делении на числа – 2 на р . Если s p-1 = 0, то число 2р – 1 простое. Так, для р = 2281 проводить вычисления приходится не более чем с семизначными числами (ибо 2280 2 = 5 198 400), а это можно сделать в самой обычной компьютерной арифметике. Ситуация вполне аналогичная той, которая была у нас при решении задачи 1, — вместо того, чтобы искать 58-разрядное число при K = 6, мы подобрали иную форму записи такого числа и обошлись вычислениями не более чем с двузначными числами.

§ 25. Математика компьютерной арифметики

Уже в предыдущем параграфе мы несколько раз упомянули о трудностях компьютерных вычислений, связанных с ограниченностью разрядной сетки. Что стоит за этими словами?

Физически память компьютера — это пронумерованная совокупность ячеек. В свою очередь, каждая ячейка состоит из восьми устройств, каждое из которых может находиться в одном из двух состояний. Одно из состояний кодируется символом 0, а другое — символом 1. Тем самым, в одной ячейке размещается 1 байт информации.

Содержимое каждой ячейки, если отбросить возможно стоящие в ней первые нули, можно рассматривать как запись натурального числа в двоичной системе счисления. При этом коду 00000000 сопоставляется число 0.

Однако, кроме положительных целых чисел, существуют отрицательные. Чтобы указать знак числа, нужен еще один бит. При этом договариваются, что его нулевое значение соответствует знаку “+”, а единичное значение — знаку “–”. Обычно под знак выделяют самый левый бит ячейки. Тогда код наибольшего натурального числа, которое можно записать в одной ячейке, — это 01111111. Ему соответствует десятичное число +127. А код наименьшего отрицательного числа — это 111111111; в десятичной записи ему соответствует число –127.

Чтобы найти сумму чисел 101 2 и 1011 2 , с их кодами можно действовать по правилам сложения двоичных чисел: 00000101 + 00001011 = 00010000. Но попытка по тем же правилам сложить коды чисел 1010101 2 и 111101 2 приведет к странному результату: 01010101 + 00111101 = 100100010. Результат оказался отрицательным числом!

Каждому ясно: ошибка возникла из-за того, что разрядная сетка ячейки содержит всего семь мест. Впрочем, если ячейка имела бы и больше разрядов, все равно такая ошибка возникнет каждый раз, как потребуется найти сумму достаточно больших чисел.

Описанное выше явление называется эффектом переполнения . И о нем приходится помнить, когда имеешь дело с достаточно большими целыми числами.

Попытаемся теперь сложить положительное число с отрицательным. Например, число 1010 2 с числом –101 2 . Их коды, соответственно, таковы: 00001010 и 10000101. А результатом должно быть натуральное число 101 2 . Даже сам алгоритм получения кода результата сформулировать не так уж просто (попытайтесь, ради интереса, такое проделать), а тем более реализовать простое устройство, которое бы этот алгоритм исполняло. А ведь сложение компьютеру приходится выполнять довольно часто, так что все должно быть как можно проще и быстрее.

Для того чтобы выполнение операции сложения было проще и не зависело от знака слагаемых, отрицательные целые числа кодируют другим способом. Но сначала поговорим о нуле.

Что будет, если в восьмиразрядную ячейку попытаться записать натуральное число 100000000 2 ? Все восемь разрядов окажутся нулями. Значит, компьютер воспринимает это число как 0. Этим-то мы и воспользуемся. Вычтем из числа 100000000 2 число 101 2 . Получится 11111011 2 . Если теперь это число сложить с числом 101 2 , то компьютер воспримет результат как 0. Поэтому число 11111011 2 естественно объявить кодом отрицательного числа –101 2 . Его называют дополнительным кодом данного отрицательного числа.

Об одном коде поговорим отдельно. Рассмотрим код 10000000. Во-первых, это код отрицательного числа, поскольку в самом левом разряде стоит 1. Компьютер воспринимает его как дополнительный код. Тогда прямой код противоположного ему положительного числа совпадает с разностью 100000000 2 – 10000000 2 . Она равна 10000000, т.е. 128 10 . Таким образом, наименьшее отрицательное число, которое можно записать в восьмибитовую ячейку, — это –128.

Кодом числа 0 является 00000000. А что произойдет, если записать дополнительный код для числа –0? Вычитаем из 100000000 2 число 0 2 и получаем 10000000 2 . При записи в ячейку снова окажется 00000000. Так что компьютер, как и мы с вами, не различает числа +0 и –0.

Конечно, для вычисления дополнительного кода отрицательного числа n можно каждый раз вычитать из 100000000 2 модуль числа n . Но есть и другой способ. Вот как можно поступать, чтобы получить дополнительный код отрицательного числа:

1. Записать двоичный код модуля числа.

2. В полученной записи каждую цифру 1 заменить цифрой 0, а каждую цифру 0 — цифрой 1.

3. К полученному коду, рассматриваемому как натуральное число в двоичной системе, прибавить 1.

Как видите, построение дополнительного кода осуществляется легко. И поскольку вычитание можно заменить сложением с противоположным числом, переход к дополнительному коду числа позволяет вообще обойтись без вычитания.

Конечно, диапазон от –128 до +127 во многих случаях маловат для решения возникающих задач. Но вовсе не обязательно хранить целое число ровно в одной восьмибитовой ячейке. Обычно для целых чисел отводится две ячейки — именно так происходит, если вы объявляете тип переменной Integer. Если же объявить тип LongInt, то под целое число будут отведены 4 восьмибитовые ячейки. А вот одна ячейка отводится, если объявлен тип ShortInt.

Если под запись числа отведено две ячейки, то они воспринимаются как единое целое. Принцип же кодирования тот же: самый левый разряд отводится под знак числа, остальные пятнадцать разрядов — для кода абсолютной величины числа. Для положительных чисел используется прямой код , для отрицательных — дополнительный. Тем самым, диапазон целых чисел таков: от –32 768 до +32 767 = 2 15 – 1.

Вообще, если для записи целых чисел используется m -битный двоичный код, то диапазон кодируемых чисел от –2 m –1 до 2 m –1 – 1, при этом дополнительный код отрицательного числа n из этого диапазона совпадает с записью в двоичной системе числа 2 m + n .

Ясно, что при умножении целых чисел эффект переполнения возникает еще чаще. А как на это реагирует компьютер? Переполнение при выполнении операций с числами, записанными с фиксированной запятой, не вызывает прерывания работы процессора . В зависимости от используемого языка диагностика может отсутствовать (так, например, происходит при использовании большинства версий языка Турбо Паскаль), может сообщать пользователю о данном событии и ждать его реакции, а может просто переходить к записи числа с плавающей запятой (так бывает в различных версиях языка Бейсик).

Конечно, кроме целых чисел, человек активно пользуется дробями. И любое вещественное число может быть записано конечной или бесконечной десятичной дробью. Впрочем, такое представление нужно бывает, как правило, лишь в теоретических построениях. А на практике…

На практике в большинстве случаев приходится иметь дело именно с приближенными значениями величин. Приближенные значения возникают при измерениях, при подсчете больших величин, да и во многих других случаях. Поэтому исследователь или инженер, решая ту или иную задачу, должен изначально оценить, сколько значащих цифр следует иметь в ходе проводимых им вычислений и сколько оставить в результате. Но об этом речь обычно идет в курсе математики. Мы же сейчас хотим обратить ваше внимание на другое.

Допустим, решено, что достаточно трех значащих цифр. Тогда в числах, скажем, 37 200 000; –372; 3,72; 0,000372 нам требуется знать лишь эти три цифры и то, начиная с какого десятичного разряда они записаны. Именно такую информацию и надо сообщить компьютеру. Для этого представим числа единообразно: пишем 0 (а перед ним знак “–”, если число отрицательно), затем запятую, сразу после запятой пишем значащие цифры и получившуюся десятичную дробь умножаем на подходящую степень числа 10. Вот что получится для четырех указанных выше чисел:

37 200 000 = 0,372 ·10 8 ;

–372 = –0,372 · 10 3 ;

3,72 = 0,372 · 10 1 ;

0,000372 = 0,372 · 10 –3 .

Ясно, что абсолютную величину любого числа можно представить как произведение числа, заключенного между 0,1 и 1, и степени числа 10 с целым показателем. Дробная часть первого множителя в таком представлении называется мантиссой числа, а показатель степени числа 10 — порядком числа. Само представление числа в виде такого произведения называется нормализованной записью числа. По-другому такое представление чисел называют записью чисел с плавающей запятой . Максимально допустимое количество разрядов в мантиссе числа определяет точность, с которой данное число может быть представлено.

В компьютере числа представлены в двоичной системе счисления. Для двоичной системы нормализованный вид числа — это представление его в виде ±m ·2 p , где 0,1 2 m р — целое число. Например,

0,1 10 = 0,0(0011) 2 = 0,11(0011) · 2 –3 .

Число с плавающей запятой в памяти компьютера может занимать разное количество байтов. Для записи числа, как говорят, с обычной точностью отводится 4 байта, а числа двойной точности занимают 8 байтов. Впрочем, в зависимости от конструкции компьютера встречаются и другие варианты, например, когда под запись числа отводится 10 байтов.

В большинстве практических случаев того, что называют обычной точностью представления действительных чисел, оказывается вполне достаточно. Поэтому и мы дальше рассмотрим только этот случай. В нем под знак числа и порядок отводится
1 байт, и это первые 8 разрядов. Остальные 24 разряда отводятся под мантиссу. Это, например, означает, что мантисса компьютерного представления числа 0,1 10 будет такова:

110011001100110011001101

Знак числа кодируется обычным образом: “плюс” кодируется символом 0, “минус” — символом 1. Что касается порядка числа, то там записывается так называемый машинный порядок . Под него отведено семь разрядов, и он равен целому неотрицательному числу, для которого данный код является прямым двоичным кодом. Например, коду 0101011 соответствует машинный порядок 43. Машинный порядок связан с порядком числа следующим образом:

порядок числа = машинный порядок – 2 6 .

Тем самым, цепочка 0101011 является кодом порядка, равного –21. А последовательность 0000000 кодирует порядок –64. Нетрудно видеть, что нулевой порядок кодируется как 1000000. Наибольший положительный порядок имеет код 1111111 и равен 63.

Вот как выглядит полный компьютерный код числа 0,1 10:

Заметим еще раз, что представление чисел с плавающей запятой в памяти компьютера может осуществляться весьма по-разному — в зависимости от языка программирования, особенностей архитектуры
и т.д. Например, в некоторых языках под мантиссу отводится всего лишь один байт и знак числа записывается при мантиссе, а не выносится в крайний левый разряд. Мы здесь продемонстрировали лишь основные принципы представления таких чисел.

Рассмотрим теперь сложение чисел с плавающей запятой. Если у чисел в нормализованном виде порядки одинаковы, то достаточно сложить мантиссы этих чисел, а затем нормализовать полученный результат, если эта сумма окажется больше 1 или меньше 0,1. Например,

Хотя примеры приведены в двоичной системе счисления, ясно, что высказанное правило действует в любой позиционной системе.

Если же порядки нормализованных чисел различны, то при сложении чисел с плавающей запятой предварительно выполняется операция выравнивания порядков слагаемых. Заметим, что величина числа не меняется при сдвиге мантиссы на один разряд вправо с одновременным увеличением порядка на 1. Поэтому в том слагаемом, у которого порядок меньше, производится сдвиг мантиссы вправо на разность между порядками, после чего мантиссы складываются, и результат снова нормализуется. Например,

Значит, можно сформулировать следующее правило сложения чисел с плавающей запятой.

Намного проще выполняется умножение и деление чисел в нормализованном виде.

Например,

0,101 · 2 –3 · 0,1011 · 2 4 = (0,101 · 0,1011) · 2 –3+4 = 0,0110111 · 2 1 = 0,110111 · 2 0 .

Например,

0,1011 · 2 –3: 0,101 · 2 4 = (0,1011: 0,101) · 2 –3–4 = 1,0001100110011… · 2 –7 = =0,10001100110011… · 2 –6 .

Последний пример показывает, что на практике даже при обычных вычислениях мы будем вынуждены довольствоваться лишь приближенным значением дроби. В компьютере ограниченность разрядной сетки играет еще большую роль. Пусть, к примеру, мы складываем числа a = 0,1 · 2 13 и b = 0,1 · 2 –12 . После выравнивания порядков получаем:

Но в разрядную сетку для мантиссы помещается лишь 24 цифры, а не 26. Поэтому два последних разряда будут утеряны, и результат совпадет с первым слагаемым. Получается, что в “компьютерной арифметике” вполне может оказаться a + b = a , хотя b 0. И это далеко не единственная особенность компьютерной арифметики. В задании 15 приведен пример, показывающий, что может не выполняться сочетательный закон для сложения .

При умножении и делении нормализованных чисел одним из возможных эффектов является переполнение разрядной сетки для порядка числа. Вот пример:

0,1001 · 2 32 · 0,11 · 2 33 = 0,011011 · 2 65 = 0,11011 · 2 64 .

Но наивысший возможный порядок — это 63. Следовательно, данный результат непредставим в компьютерной арифметике. Обычно операционная система или компилятор языка программирования в таком случае дает диагностику “переполнение”. Впрочем, переполнение для нормализованных чисел может возникнуть и при сложении; в задании 17 вам предлагается построить соответствующий пример.

Кроме рассмотренных случаев, когда при вычислении на компьютере получается неверный результат или результат вообще не получается, надо иметь в виду эффекты, связанные с округлением чисел. Эти эффекты также обусловлены ограниченностью разрядной сетки. Одним из них является потеря значащих цифр. Возьмем, к примеру, числа 1/1000 и 1/1004. В двоичной системе счисления в нормализованном виде они с учетом округления равны соответственно 0,100000110001001001101111 · 2 –9 и 0,100000101000110010110000 · 2 –9 . После вычитания получится 0,1000010110111111 · 2 –17 . Точное значение разности равно 4/1004000. При записи в нормализованном виде с 24-разрядной мантиссой получаем 0,100001011010111011101100 · 2 –17 . Как видите, только первые 11 цифр после запятой оказались точными.

Итак, компьютерная арифметика может давать следующие эффекты:

1) ошибки округления, которые возникают при записи чисел с округлением и могут накапливаться при выполнении операций;

2) переполнение разрядной сетки порядка чисел приводит к получению невоспроизводимого в компьютере числа;

3) потеря значащих цифр при вычитании близких чисел или при переполнении разрядной сетки мантиссы;

4) игнорирование слагаемого при большой разнице в порядках.

В качестве практической рекомендации отметим, что для вещественных чисел, которые представляются в компьютере в нормализованном виде, не следует сравнивать их на точное равенство; вместо этого надо сравнивать абсолютную величину их разности с подходящим маленьким положительным числом, соответствующим погрешности представления.

Вопросы и задания

1. Какова основная причина различных эффектов компьютерной арифметики?

2. Что такое дополнительный код? Каковы преимущества использования дополнительного кода?

3. Каков диапазон целых чисел, для кодирования которых используются 4 ячейки?

4. В чем состоит эффект переполнения?

5. Объясните, почему приведенный в объяснительном тексте параграфа способ действительно дает дополнительный код отрицательного числа.

6. а) Следующие целые числа записаны в прямом восьмибитном коде: 01101010; 10111011; 10001001; 01010111; 11111111. Найдите среди них отрицательные числа и запишите их в дополнительном коде.

б) Следующие числа записаны в прямом шестнадцатибитном коде:

Найдите среди них отрицательные числа и запишите их в дополнительном коде.

7. а) Выполните сложение чисел 100 10 и 50 10 в однобайтовом коде представления целых чисел с фиксированной запятой. Кодом какого числа (в десятичной системе счисления) будет полученный результат? (Совет: не забудьте, что отрицательные числа представляются в дополнительном коде. )

б) Выполните сложение чисел –80 10 и –64 10 в однобайтовом коде представления целых чисел с фиксированной запятой. Кодом какого числа (в десятичной системе счисления) будет полученный результат?

8. Почему при компьютерном сложении целых чисел может нарушаться сочетательный закон
(a + b ) + c = a + (b + c )? Для обоснования ответа приведите подходящий пример.

9. Какую запись числа называют нормализованной? Что такое мантисса и порядок числа?

10. Каким будет первый символ кода машинного порядка числа, если порядок числа, представленного в двоичном нормализованном виде, неотрицателен?

11. В объяснительном тексте параграфа приведена четырехбайтовая запись числа 0,1 10 в памяти компьютера.

а) Какому десятичному числу на самом деле соответствует эта запись?

б) Каковы абсолютная и относительная погрешности представления числа 0,1 10 ?

в) Объясните, почему последней цифрой мантиссы оказалась цифра 1.

12. Укажите, какие из нижеприведенных действительных чисел могут быть представлены в памяти компьютера четырехбайтовым кодом числа с плавающей запятой, описанным в объяснительном тексте.

13. Выполните сложение двоичных чисел, представленных в нормализованном виде. Результат нормализуйте.

14. Выполните умножение двоичных чисел, представленных в нормализованном виде. Результат нормализуйте.

15. По правилам компьютерной арифметики вычислите суммы

Убедитесь, что для этих чисел нарушается сочетательный закон.

16. Объясните, почему при умножении чисел с плавающей запятой может нарушаться сочетательный закон. Приведите подтверждающие примеры.

17. Приведите пример двух нормализованных двоичных чисел, при сложении которых может произойти переполнение.

18. Для решения некоторой задачи Петя составил следующий алгоритм:

алг Сумма1 (арг цел N, рез вещ : S)

нач цел : K

ввод N

нц для K от 1 до N

вывод S

В свою очередь, Коля для решения той же задачи составил такой алгоритм:

алг Сумма2 (арг цел N, рез вещ : S)

нач цел : K

ввод N

нц для K от N до 1 с шагом –1

кц

вывод S

а) Верно ли, что оба алгоритма решают одну и ту же задачу?

б) Верно ли, что при N = 1 000 000 000 программы, реализующие данные алгоритмы, дадут одинаковый результат, если используется четырехбайтовое представление нормализованных чисел, описанное в тексте параграфа?

1 Вечно она, конечно, работать не будет — или произойдет переполнение памяти компьютера в ходе вычисления все новых и новых цифр, или Чубайс свет вырубит…

2 Этот прием достаточно хорошо известен математикам, но весьма редко используется в программировании.

3 Между прочим, доказательство этого утверждения — о том, что при других значениях K нет решений задачи 2, не совпадающих с решением задачи 1, — автор статьи без компьютера получать не умеет.

4 Впрочем, история математики так же изобилует белыми пятнами, как и любая другая история. Некоторые исследователи считают, что простыми числами Мерсенна интересовались уже пифагорейцы в связи с так называемыми “совершенными числами”.

Двоичная система счисления презентация. Все есть число…

Слайд 2

Цитаты

Всё наше достоинство заключено в мысли… Будем же учиться хорошо мыслить. Б. Паскаль Учение без размышления бесполезно, но и размышление без учения опасно. Конфуций Лучше понять немного, чем понять неверно. Л. Франс Всё, что мы знаем, — ограничено, чего мы не знаем – бесконечно. Лаплас Лучше знать лишнее, чем ничего не знать. Сенека

Слайд 3

Система счисления – совокупность приёмов и правил для обозначения чисел. Системы счисления Позиционная система счисления – система счисления, в которой одна и та же цифра получает различные количественные значения в зависимости от места, или позиции, которое она занимает в записи данного числа. Рассмотрим десятичные числа Можно предположить, что они одинаковые, так как в них участвуют одни и те же цифры – 3 и 4? Вы не согласны? Объясните почему? К позиционной системе счисления относятся десятичная система счисления и двоичная система счисления. — Позиционные — Непозиционные 43 и 34

Слайд 4

Система счисления называется непозиционной, если в ней количественные значения символов, используемых для записи чисел, не зависят от их положения (места, позиции) в коде числа. Например, в римской системе счисления запись IX обозначает число 9, а запись XI — число 11. Десятичное число 28 представляется следующим образом: XXVIII = 10+10+5+1+1+1 Десятичное число 99 имеет такое представление: XCIX = -10+100-1+10

Слайд 5

Значимость двоичной системы счисления для кодирования информации

В ЭВМ используют двоичную систему, потому что она имеет ряд преимуществ перед другими системами: для ее реализации нужны технические элементы с двумя возможными состояниями (есть ток, нет тока; включено, выключено и т. д.; одному из состояний ставится в соответствие 1, другому – 0), а не десять, как в десятичной системе; представление информации посредством только двух состояний надежно и помехоустойчиво; упрощается выполнение арифметических действий; возможность использования аппарата булевой алгебры для выполнения логических преобразований информации.

Слайд 6

Чарльз Бэббидж (1791-1871), английский математик и инженер, разработавший принципы, на основе которых конструируются все современные компьютеры. Аналитическая машина

Слайд 7

Леди-программист Августа Ада Лавлейс

Суть и предназначение машины изменятся от того, какую информацию мы в неё вложим. Машина сможет писать музыку, рисовать картины и покажет науке такие пути, которые мы никогда и нигде не видели. Ада Лавлейс Ада Лавлейс предложила Чарльзу Бэббиджу применить двоичную систему счисления. Она написала несколько программ для аналитической машины, разработала теорию программирования.

Слайд 8

Вильгельм Готфрид Лейбниц (1646-1716)

Начиная со студенческих лет и до конца жизни великий европеец, немецкий ученый Вильгельм Готфрид Лейбниц занимался исследованием свойств двоичной системы счисления, ставшей в дальнейшем основной при создании компьютеров. Изображение медали В. Лейбница

Слайд 9

10  2 2  10 19 2 9 18 1 2 4 8 1 2 2 4 0 2 1 2 0 2 0 0 1 19 = 100112 система счисления 100112 4 3 2 1 0 разряды = 1·24 +0·23+0·22+1·21+1·20 = 16 + 2 + 1 = 19 Перевод чисел 1 1 0 0 1 Системы счисления

Системы счисления. Перевод чисел из десятичной в двоичную систему счисления.

Презентация создана для учащихся 8 класса, которые только знакомятся с понятиями: система счисления, десятичная, двоичная, позиционная, непозиционная; и, которые, по моему мнению должны освоить правила перевода чисел из десятичной в двоичную СС и наоборот.

Презентация может быть использована для повторения в старших классах.


Расскажи мне, и я забуду, покажи мне, и я запомню, дай мне попробовать,

и я научусь.

Китайская мудрость


Теория

  • Все есть число… Десятичная система счисления Двоичная система счисления Чтение чисел
  • Все есть число… Определение понятия «Система счисления» Десятичная система счисления Двоичная система счисления Чтение чисел
  • Все есть число…
  • Определение понятия «Система счисления»
  • Десятичная система счисления
  • Двоичная система счисления
  • Чтение чисел

Тренировочные задания

  • Тренировочные задания
  • Тренировочные задания
  • Практика Контроль знаний
  • Перевод из десятичной СС в двоичную(теория) Практика Контроль знаний
  • Перевод из десятичной СС в двоичную(теория) Практика Контроль знаний
  • Перевод из десятичной СС в двоичную(теория)
  • Практика
  • Контроль знаний

Все есть число…

  • Люди предпочитают десятичную систему счисления вероятно потому, что с древних времен они считали по пальцам, а у людей по 10 пальцев на руках и ногах.
  • Десятичная система счисления пришла к нам из Индии.
  • Для общения с ЭВМ используют, кроме десятичной, двоичную, восьмеричную и шестнадцатеричную системы счисления.
  • Из всех систем счисления особенно проста и поэтому интересна для технической реализации в ЭВМ двоичная система счисления.

Определение понятия «Система счисления»

  • Система счисления — это способ записи чисел с помощью заданного набора специальных знаков и соответствующие ему правила выполнения действий над числами.
  • Все системы счисления делятся на две большие группы

позиционные

величина, которую обозначает цифра в записи числа, зависит от положения цифры в этом числе

непозиционные

величина, которую обозначает цифра в записи числа, не зависит от положения цифры в этом числе


Десятичная система счисления


Двоичная система счисления


Чтение чисел

  • В десятичной системе можно прочитать запись 36 – как число «тридцать шесть», запись 101 – как число «сто один» и т. д.
  • Но в других системах счисления, например, в интересующей нас двоичной, надо говорить так: запись 101 2 – число «один – ноль- один» в двоичной системе счисления.

Способ перевода числа из десятичной системы в двоичную


Тренировочные задания

  • 31, 68, 147
  • Перевести из десятичной в восьмиричную систему:
  • 5, 24, 99

Домашнее задание

  • Перевести из десятичной в двоичную систему:
  • Перевести из десятичной в восьмиричную систему – заполнить таблицу.

Запомни

2 0

2 1

2 2

2 3

2 4

2 5

2 6

2 7

2 8

2 9

2 10

Слон живет у нас в квартире,

В доме два, подъезд четыре.

По часам привык питаться –

Утром в восемь, днем в шестнадцать.

Съест на завтрак непременно

Тридцать две охапки сена,

После утренней прогулки –

Шестьдесят четыре булки.

На обед ему приносим

Огурцов сто двадцать восемь.

Помидоров может съесть

Двести пятьдесят и шесть,

Съесть блинов пятьсот двенадцать,

Это если не стараться.

А замесишь на кефире –

Тысячу двадцать четыре.


Контроль знаний

1.Перевести из десятичной системы счисления в двоичную : 6 3 , 256, 457, 845

2.Приведите в соответствие :

1.Базис 2.Основание 3.Алфавит

А.множество символов Б.вес разряда В.размер алфавита

3.Шуточная задача:

П рилетел как-то к земной девушке, красавице писаной, ухажер с планеты

Onezero ; давай замуж ее звать и похваляться, что и зарабатывает он

1100000 долларов в месяц и апартаменты у него общей площадью

10100 кв. м., и одних машин у него 10 штук.

Однако девица наша была с умом и учла, что все это в двоичной системе.

А сколько же по-нашему будет?


Взаимопроверка

1. 63 10 = 111111 2

256 10 = 100000000 2

457 10 = 111001001 2

845 10 = 1101001101 2

3. 1100000 2 = 96 10

10100 2 = 20 10

10 2 = 2 10

Обратить внимание учащихся, что

1. если число, которое мы переводим из десятичной в двоичную систему счисления равно 2 n — 1, тогда ответ будет равен n- единиц, например,

31=32-1 =2 5 -1,т.е. не выполняя никаких вычислений, при переводе числа 31 из десятичной в двоичную СС, мы можем сразу же записать ответ: 31 10 = 11111 2

2. если число, которое мы переводим из десятичной в двоичную систему счисления равно 2 n , тогда ответ будет равен 1 и n нулей, например,

512=2 9 ,т. е. не выполняя никаких вычислений, при переводе числа 512 из десятичной в двоичную СС, мы можем сразу же записать ответ: 512 10 = 1000000000 2

1 слайд

2 слайд

* Двоичное кодирование в компьютере Вся информация, которую обрабатывает компьютер должна быть представлена двоичным кодом с помощью двух цифр: 0 и 1. Эти два символа принято называть двоичными цифрами или битами. С помощью двух цифр 0 и 1 можно закодировать любое сообщение. Это явилось причиной того, что в компьютере обязательно должно быть организованно два важных процесса: кодирование и декодирование. Кодирование – преобразование входной информации в форму, воспринимаемую компьютером, т.е. двоичный код. Декодирование – преобразование данных из двоичного кода в форму, понятную человеку. *

3 слайд

* Двоичная система счисления Двоичная система счисления — позиционная система счисления с основанием 2. Используются цифры 0 и 1. Двоичная система используется в цифровых устройствах, поскольку является наиболее простой и удовлетворяет требованиям: Чем меньше значений существует в системе, тем проще изготовить отдельные элементы. Чем меньше количество состояний у элемента, тем выше помехоустойчивость и тем быстрее он может работать. Простота создания таблиц сложения и умножения — основных действий над числами *

4 слайд

* Соответствие десятичной и двоичной систем счисления Количество используемых цифр называется основанием системы счисления. При одновременной работе с несколькими системами счисления для их различения основание системы обычно указывается в виде нижнего индекса, который записывается в десятичной системе: 12310 — это число 123 в десятичной системе счисления; 11110112 — то же число, но в двоичной системе. Двоичное число 1111011 можно расписать в виде: 11110112 = 1*26 + 1*25 + 1*24 + 1*23 + 0*22 + 1*21 + 1*20. p=10 0 1 2 3 4 5 6 7 8 9 10 11 12 13 14 15 16 p=2 0 1 10 11 100 101 110 111 1000 1001 1010 1011 1100 1101 1110 1111 10000 *

5 слайд

* Перевод чисел из одной системы счисления в другую Перевод из десятичной системы счисления в систему счисления с основанием p осуществляется последовательным делением десятичного числа и его десятичных частных на p, а затем выписыванием последнего частного и остатков в обратном порядке. Переведем десятичное число 2010 в двоичную систем счисления (основание системы счисления p=2). В итоге получили 2010 = 101002. *

6 слайд

* Перевод чисел из одной системы счисления в другую Перевод из двоичной системы счисления в систему счисления с основанием 10 осуществляется последовательным умножением элементов двоичного числа на 10 в степени места этого элемента при учете что нумерация мест идет справа и начинается с цифры «0». Переведем двоичное число 100102 в десятичную систем систем счисления. В итоге получили 100102 = 1810. 100102=1*24+ 0*23 +0*22+1*21+ 0*20 =16+2=1810 *

План-конспект урока по информатике в 9 классе на тему « Двоичная система счисления» (Слайд 1)

Цель: сформировать понятия «двоичная система счисления» и основ арифметических вычислений в двоичной системе. (Слайд 2)

Требования к знаниям и умениям (Слайд 3)

Учащиеся должны знать:

    десятичную и двоичную системы счисления;

    развернутую форму записи числа;

    правила перевода из двоичной системы счисления в десятичную и наоборот;

    правила сложения и умножения двоичных чисел.

Учащиеся должны уметь:

    переводить двоичные числа в десятичную систему;

    переводить десятичные числа в двоичную систему;

    складывать и умножать двоичные числа.

Программно-дидактическое оснащение: Сем., § 16, с. 96; демонстрация «Двоичная система счисления»; проектор. (Слайд 4)

Ход урока

    Организационный момент

    Постановка целей урока

С какими числами работает компьютер? Почему?

Как ими оперировать?

    Работа по теме урока

(С помощью демонстрации «Двоичная система счисления» показать развернутую форму числа, перевод из двоичной системы счисления в десятичную и наоборот, арифметику двоичных чисел.)

Двоичная система счисления является основной системой представления информации в памяти компьютера. Эта идея принадлежит Джону фон Нейману (Слайд 5) , сформулировавшему в 1946 г. принципы устройства и работы ЭВМ. Но, вопреки распространенному заблуждению, двоичная система счисления была придумана не инженерами-конструкторами электронных вычислительных машин, а математиками и философами, задолго до появления компьютеров, еще в XVII-XIX вв. Великий немецкий ученый Лейбниц (Слайд 6) считал: «Вычисление с помощью двоек является для науки основным и порождает новые открытия… При сведении чисел к простейшим началам, каковы 0 и 1, везде появляется чудесный порядок». Позже двоичная система была забыта, и только в 1936-1938гг американский инженер и математик Клод Шеннон (Слайд 7) нашел замечательные применения двоичной системы при конструировании электронных схем.

А что же такое система счисления? Это правила записи чисел и связанные с ними способы выполнения вычислений.

Система счисления, к которой мы все привыкли, называется десятичной. Объясняется это название тем, что в ней используются десять цифр: 0, 1,2, 3,4, 5, 6, 7, 8,9. (Слайд 8) Число цифр определяет основание системы счисления. Если число цифр — десять, то основание системы счисления равно десяти. В двоичной же системе существует всего две цифры: 0 и 1. Основание равно двум. Возникает вопрос, можно ли с помощью всего двух цифр представить любую величину. Оказывается, можно!

Развернутая форма записи числа (Слайд 9)

Вспомним принцип записи чисел в десятичной системе счисления. Значение цифры в записи числа зависит не только от самой цифры, но и от места расположения этой цифры в числе (говорят: от позиции цифры). Например, в числе 555 первая справа цифра обозначает: три единицы, следующая — три десятка, следующая — три сотни. Этот факт можно выразить как сумму разрядных слагаемых:

555 10 = 5 х 10 2 + 5 х 10 1 + 5 х 10° = 500 + 50 + 5.

Таким образом, с продвижением от цифры к цифре справа налево «вес» каждой цифры увеличивается в 10 раз. Это связано с тем, что основание системы счисления равно десяти.

Перевод двоичных чисел в десятичную систему

А вот пример многозначного двоичного числа: 111011 2 . Двойка внизу справа указывает на основание системы счисления. Это нужно для того, чтобы не перепутать двоичное число с десятичным. Ведь существует же десятичное число 111011! Вес каждой следующей цифры в двоичном числе при продвижении справа налево возрастает в 2 раза. Развернутая форма записи данного двоичного числа выглядит так:

111011 2 = 1 х 2 5 + 1 х 2 4 + 1 х 2 3 + 0х 2 2 + 1 х 2 1 + 1 х 2° = 67 10 .

Таким способом мы перевели двоичное число в десятичную систему.

Переведем в десятичную систему еще несколько двоичных чисел (Слайд 10).

10 2 = 2 1 =2; 100 2 = 2 2 = 4; 1000 2 = 2 3 = 8;

10000 2 = 2 4 = 16; 100000 2 = 2 5 = 32 и т. д.

Таким образом, получилось, что двузначному десятичному числу соответствует шестизначное двоичное! И это характерно для двоичной системы: быстрый рост количества цифр с увеличением значения числа.

Задание 1. (Слайд 11) Запишите начало натурального ряда чисел в десятичной (А 10 ) и двоичной (А 2 ) системах счисления.

Задание 2. Переведите в десятичную систему следующие двоичные числа.

101 ; 11101 ; 101010 ; 100011 ; 10110111011 .

Ответ: 5; 29; 42; 35; 1467.

Перевод десятичных чисел в двоичную систему (Слайд 12)

Как перевести двоичное число в равное ему десятичное, вам должно быть понятно из рассмотренных выше примеров. А как осуществить обратный перевод: из десятичной системы в двоичную? Для этого нужно суметь разложить десятичное число на слагаемые, представляющие собой степени двойки. Например:

15 10 = 8 + 4 + 2 + 1 = 1 х 2 3 + 1 х 2 2 + 1 х 2 1 + 1 х 2° = 1111 2 . Это сложно. Есть другой способ, с которым мы сейчас и познакомимся.

Пусть нужно перевести в двоичную систему счисления число 234. Будем делить 234 последовательно на 2 и запоминать остатки, не забывая и про нулевые:

234 = 2 х 117 + 0 14 = 2 х 7 + 0

Выписав все остатки, начиная с последнего, получим двоичное разложение числа: 234 10 = 11101010 2 .

Задание 3. (Слайд 13) Какие двоичные числа соответствуют следующим десятичным числам?

2; 7; 17; 68; 315; 765; 2047.

Ответ: 10 2 ; 111 2 ; 10001 2 ; 1000100 2 ; 100111011 2 ; 1011111101 2 ; 11111111111 2 .

Арифметика двоичных чисел (Слайд 14)

Правила двоичной арифметики гораздо проще правил десятичной арифметики. Вот все возможные варианты сложения и умножения однозначных двоичных чисел:

0+0=0

0+1=1

1+0=1

1+1=10

0*0=0

0*1=0

1*0=0

1*1=1

Своей простотой и согласованностью с битовой структурой компьютерной памяти двоичная система счисления и привлекла изобретателей компьютера. Ее гораздо проще реализовать техническими средствами, чем десятичную систему.

Вот пример сложения столбиком двух многозначных двоичных чисел (Слайд 15) :

+ 1011011101

111010110

10010110011

А теперь посмотрите внимательно на следующий пример умножения многозначных двоичных чисел:

х 1101101

101

1101101

1101101

1000100001

Задание 4. (Слайд 16) Выполните сложение в двоичной системе счисления. 11 + 1; 111 + 1; 1111 + 1; 11111 + 1.

Ответ: 100; 1000; 10000; 100000.

Задание 5. Выполните умножение в двоичной системе счисления.

111 х 10; 111 х 11; 1101 х 101; 1101 х 1000.

Ответ: 1110; 10101; 1000001; 1101000.

    Подведение итогов урока (Слайд 17)

Система счисления — это определенные правила записи чисел и связанные с этими правилами способы выполнения вычислений. Основание системы счисления равно количеству используемых в ней цифр.

Двоичные числа — это числа в двоичной системе счисления. В их записи используются две цифры: 0 и 1.

Развернутая форма записи двоичного числа — это его представление в виде суммы степеней двойки, умноженных на 0 или на 1.

Использование двоичных чисел в компьютере связано с битовой структурой компьютерной памяти и с простотой двоичной арифметики

Домашнее задание (Слайд 18)

    Заданы двоичные числа Х и Y . Вычислить X + Y и X- Y , если Х= 1000111, Y = 11010.

    Заданы двоичные числа X и У. Вычислить X + Y 1001101, если X = 1010100, Y = 110101.

    Выполнить умножение: 100110 х 11001.

Ответы: 1.1100001 и 101101; 2. 111100; 3. 1110110110.

* Тест «Системы счисления» — Электронные облака

Тест на знание следующих областей:

  • Перевод чисел из 10СС в 2СС
  • Перевод чисел из 2СС  в 10СС
  • Решение задачи с данными, представленными в различных СС
  • Настройки приложения Калькулятор
  • Перевод чисел из 2СС в 10СС и построение по полученным координатам рисунка на координатной плоскости

1. Что называется системой счисления?

2. На какие два типа можно разделить все системы счисления?

3. Какие системы счисления называются непозиционными? Почему? Приведите пример такой системы счисления и записи чисел в ней?

4. Какие системы счисления применяются в вычислительной технике: позиционные или непозиционные? Почему?

5. Какие системы счисления называются позиционными?

6. Как изображается число в позиционной системе счисления?

7. Что называется основанием системы счисления?

8. Что называется разрядом в изображении числа?

9. Как можно представить целое положительное число в позиционной системе счисления?

10. Приведите пример позиционной системы счисления.

11. Опишите правила записи чисел в десятичной системе счисления: 
а) какие символы образуют алфавит десятичной системы счисления? 
б) что является основанием десятичной системы счисления? 
в) как изменяется вес символа в записи числа в зависимости от занимаемой позиции?

12. Какие числа можно использовать в качестве основания системы счисления?

13. Какие системы счисления применяются в компьютере для представления информации?

14. Охарактеризуйте двоичную систему счисления: алфавит, основание системы счисления, запись числа.

15. Почему двоичная система счисления используется в информатике?

16. Дайте характеристику шестнадцатеричной системе счисления: алфавит, основание, запись чисел. Приведите примеры записи чисел.

17. По каким правилам выполняется сложение двух положительных целых чисел?

18. Каковы правила выполнения арифметических операций в двоичной системе счисления?

19. Для чего используется перевод чисел из одной системы счисления в другую?

20. Сформулируйте правила перевода чисел из системы счисления с основанием р в десятичную систему счисления и обратного перевода: из десятичной системы счисления в систему счисления с основанием S. Приведите примеры.

21. В каком случае для перевода чисел из одной системы счисления (СС) в другую может быть использована схема Горнера вычисления значения многочлена в точке? Каковы преимущества ее использования перед другими методами? Приведите пример.

22. Как выполнить перевод чисел из двоичной СС в восьмеричную и обратный перевод? Из двоичной СС в шестнадцатеричную и обратно? Приведите примеры. Почему эти правила так просты?

23. По каким правилам выполняется перевод из восьмеричной в шестнадцатеричную СС и наоборот? Приведите примеры.

(с) http://inf.e-alekseev.ru/text/Vopr3.html

———————-

Тест по системам счисления

  • Знаковая система, в которой числа записываются по определённым правилам с помощью знаков некоторого алфавита, называемых цифрами — это …

Ответ: система счисления

  • Система счисления, в которой количественное значение цифры зависит от её положения в числе, называется …

Ответ: позиционной

  • Установите соответствие между система счисления

1) позиционная система счисления
2) непозиционная система счисления
3) алфавитная система счисления
4) знаковая система счисления
5) разрядная система счисления

А) Шестнадцатеричная система счисления
Б) Зарубки, используемые Робинзоном Крузе для подсчёта времени
В) Двоичная система счисления
Г) Вавилонская система счисления
Д) Римская система счисления

Ответ: А1; Б2; В1; Г1; Д2.

  • Какое минимальное основание может иметь система счисления, если в ней записано число:

1) 16-ая система счисления
2) 7-ая система счисления
3) 8-ая система счисления
4) 2-ая система счисления
5) 10-ая система счисления

А) А7
Б) 11
В) 507
Г) 356
Д) 99

Ответ: А1; Б4; В3; Г2; Д5.

  • Системы счисления делятся на:
  1. позиционные и непозиционные;
  2. однозначные и неоднозначные;
  3. цифровые и буквенные;
  4. целые и дробные.
  • Основанием позиционной системы счисления называется:
  1. основание логарифма из формулы перевода чисел в системе;
  2. количество правил вычисления в системе;
  3. целая часть чисел;
  4. число отличных друг от друга знаков, которые используются для записи чисел.
  • В римской системе счисления число имеет следующее представление MMDCCXLIX представьте его в десятичном виде:
  1. 2749
  2. 2849
  3. 2854
  4. 225249
  5. 1749
  • Число 1785 имеет следующее представление в римской системе счисления:
  1. MCCCDLXXXV
  2. МDCCLXXXV
  3. MMCCCLXXXV
  4. МDCCLLXXV
  5. МDCLXXXXV
  • Укажите запись числа 3,1910 в развёрнутой форме:
  1. 3*101+1*100+9*10-1
  2. 3*31+1*30+9*3-1
  3. 3*100+1*10-1+9*10-2
  4. 3*21+1*20+9*2-1
  5. 3*20+1*2-1+9*2-2
  • Укажите запись числа BAD716 в развёрнутой форме:
  1. B*162+A*161+D*160+7*16-1
  2. B*160+A*161+D*162+7*163
  3. B*163+A*162+D*161+7*160
  4. B*164+A*163+D*162+7*161
  5. B*161+A*162+D*163+7*164
  • Укажите запись числа 11,012 в развёрнутой форме:
  1. 1*22+1*21+0*20+1*2-1
  2. 1*101+1*100+0*10-1+1*10-2
  3. 1*161+1*160+0*16-1+1*16-2
  4. 1*81+1*80+0*8-1+1*8-2
  5. 1*21+1*20+0*2-1+1*2-2
  6. 1*22+1*21+0*2-1+1*2-2
  • XL- не позиционная система счисления, как это число выглядит в позиционной системе счисления?
  1. 11
  2. 40
  3. 60
  4. 90
    Двоичное число 100012 соответствует десятичному числу
  1. 1110
  2. 1710
  3. 25610
  4. 100110
  5. 1000110
    Число 248 соответствует числу
  1. 1011016
  2. 2016
  3. 7616
  4. BF16
  5. 1416
  • Сообщение, записанное буквами 64-хсимвольного алфавита, содержит 20 символов. Какой объем информации оно несет?
  1. 20 байт
  2. 1280 бит
  3. 120 бит
  4. 180 бит
  • Определить сумму трех чисел: 0012 + 0178 + 1112
  1. 111012
  2. 0358
  3. 02510
  4. 101112
  • Умножьте числа в двоичной системе счисления: 110112*1012.
  • Дано:  a=9D16, b=2378. Какое из чисел С, записанных в двоичной системе счисления, удовлетворяет неравенству a<C<b ?
  1. 100110102
  2. 100111102
  3. 100111112
  4. 110111102
  • Найдите десятичное число x, такое что 20 < x < 30, запись которого в системе счисления с основанием 3  заканчивается на 11.

Ответ: 22

  • Для 5 букв латинского алфавита заданы их двоичные коды (для некоторых букв — из двух бит, для некоторых — из трех). Эти  коды представлены в таблице:

a            b      c        d        e
000    110    01    001    10
Определите, какой набор букв закодирован двоичной строкой 1100000100110

  1. baade
  2. badde
  3. bacde
  4. bacdb
  • Какое количество цифр используется в десятичной системе счисления?
  1. 8
  2. 11
  3. 10
  4. 2
    Сколько цифр в двоичной системе счисления
  1. 1
  2. 0
  3. 2
  4. 3
    В какой системе счисления хранится информация в компьютере?
  1. Троичной
  2. Двоичной
  3. Десятичной
  4. Двенадцатиричной
  • Какое число лишнее:
  1. FF16
  2. 22610
  3. 3778
  4. 111111112
  • Укажите самое большое число:
  1. 14416
  2. 14410
  3. 1448
  4. 1446
  • Переведите число 101010101 из двоичной системы счисления в десятичную
  1. 361
  2. 564
  3. 455
  4. 341
  • Каков результат перевода числа 216 из десятичной системы счисления в двоичную?
  1. 11001100
  2. 11011000
  3. 11100000
  4. 11001000
  •   Число 32в десятичной системе эквивалентно числу в двоичной системе. ..
  1. 100000
  2. 35
  3. 21
  4. 1000
  • Десятичное число 37 в двоичной системе счисления будет равно …
  1. 100101
  2. 1100101
  3. 100111
  4. 1000000
  • Чему равна сумма чисел 1916 и 318?
  1. 1110012
  2. 5010
  3. 508
  4. 5016
  • Переведите дробное число 768,54 из десятичной системы счисления в двоичную.
  • Переведите дробное число 195,22 из десятичной системы счисления в восьмеричную.
    •   Какое из приведенных ниже равенств верно?
    1. 5 в десятичной системе = 00000111 в двоичной
    2. 47 в десятичной системе = 101111 в двоичной
    3. 13 в десятичной системе = 00011111 в двоичной
    4. 2 в десятичной системе = 00001000 в двоичной
    • УКАЖИТЕ НЕПОЗИЦИОННЫЕ СИСТЕМЫ СЧИСЛЕНИЯ:
    1. Единичная.
    2. Двоичная.
    3. Римская.
    4. Десятичная.
    5. Древнеегипетская.
    6. Восьмеричная.
    • УСТАНОВИТЕ СООТВЕТСТВИЕ МЕЖДУ ЧИСЛАМИ, ЗАПИСАННЫМИ В ДВОИЧНОЙ СИСТЕМЕ И ЧИСЛАМИ, ЗАПИСАННЫМИ В ДЕСЯТИЧНОЙ СИСТЕМЕ СЧИСЛЕНИЯ.

    1.11012
    A.1910

    2.1012
    B.1310

    3.100112
    C.510

    • В классе 1111002% девочек и 11002 мальчиков. Сколько учеников в классе?

    Ответ: 30
    • В классе 1100102% девочек и 11112 мальчиков. Сколько учеников в классе?
    Ответ: 30
    • В саду 100x фруктовых деревьев, из которых 21x яблони, 22x груши, 16x слив; 17x вишен. Укажите основание системы счисления (чему равен x?)
    1. 4
    2. 6
    3. 8
    4. 10
    • Дано: a = DD16, b = 3378. Какое из чисел С, записанных в двоичной системе счисления, удовлетворяет неравенству a < C < b?
    1. 110110102
    2. 111111102   
    3. 110111102
    4. 110111112
    • Запись числа 658 в некоторой системе счисления выглядит так: 311q. Найдите основание системы счисления q.
    Ответ: 4
    • Найдите основание системы счисления, в которой выполнено сложение: 144 + 24 = 201.
    Ответ: 7
    • Для кодирования сообщения, состоящего только из букв А, Б, В и Г, используется неравномерный по длине двоичный код:
     А Б В Г
     0011
    010
    011
    Если таким способом закодировать последовательность символов ВГАГБВ и записать результат в шестнадцатеричном коде, то получится:
    1. CDADBC16
    2. A7C416
    3. 41271016
    4. 4С7А16
    • По умолчанию в приложении Калькулятор активизирован пе­реключатель Dec, означающий работу в …
    1. шестнадцатеричной системе счисления
    2. десятичной системе счисления
    3. восьмеричной системе счисления
    4. двоичной системе счисления
    • В двоичную систему счисления приложение Калькулятор пере­водится переключателем . ..
    1. Hex
    2. Dec
    3. Oct
    4. Bin
    • Отметьте и последовательно соедините на координатной плоскости точки, координаты которых приведены в двоичной системе счисления. но сначала переведите все числа в десятичную систему счисления. (кошка)
     №п/п 2 СС
    10 СС
     №п/п  2 СС 10 СС  №п/п 2 СС 10 СС
     1(011, 001)  14(100, 1001)  27(1011, 10000)  
     
     2(011, 010)  15(101, 1001)  28(1011, 110) 
     3(010, 010)  16(101, 1010)  29(1001, 110) 
     4(010, 011)  17(110, 1010)  30(1001, 100) 
     5(001, 011)
      18(110, 1011)
      31(1110, 100)
     
     6(001, 100)  19(111, 1011)
      32(1110, 011)
     
     7(1000, 100)
      20(111, 1100)
      33(1101, 011) 
     8(1000, 110)
      21(1000, 1100)
      34(1101, 010) 
     9(010, 110)
      22(1000, 1101)
      35(1100, 010) 
     10(010, 111)
      23(1001, 1101)  36(1100, 001)
     
     11(011, 111)
      24(1001, 1110)
      37(011, 001)
     
     12(011, 1000)
      25(1010, 1110)
        
     13(100, 1000)  26(1010, 10000)
        

    Перевод координатных точек рисунка из 2СС в 10СС:

     Y                
                     
                     
                     
     17                
     16                
     15                
     14                
     13                
     12                
     11                
     10                
     9                
     8                
     7                
     6                
     5                
     4                
     3                
     2                
     1                
     0 12
    3
    4
    5
    6
    7
    8
    9
    10
    11
    12
    13
    14
     15 16

    • Отметьте и последовательно соедините на координатной плоскости точки, координаты которых приведены в двоичной системе счисления. но сначала переведите все числа в десятичную систему счисления. (корабль)
     №п/п 2 СС
    10 СС
     №п/п  2 СС 10 СС  №п/п 2 СС 10 СС
     1(011, 001)
      14(1000, 111)
      27(1010, 101)
     
     2(011, 110)  15(1001, 111)  28(1010, 100)
     
     3(010, 110)  16(1010, 1010)  29(1001, 100)
     
     4(010, 1011)  17(1011, 1010)  30(1001, 010)
     
     5(011, 1011)  18(1011, 1000)  31(1010, 010) 
     6(011, 111)  19(1101, 1000)  32(1010, 001) 
     7(100, 111)  20(1101, 1010)  33(1000, 001) 
     8(100, 1000)  21(1110, 1010)  34(1000, 100) 
     9(101, 1000)  22(1110, 101)  35(100, 100) 
     10(101, 1001)  23(1101, 101)  36(100, 010) 
     11(111, 1001)  24(1101, 100)  37(101, 010) 
     12(111, 1000)  25(1011, 100)  38(101, 001)
     
     13(1000, 1000)  26(1011, 101)  39 (011, 001) 

    Перевод координатных точек рисунка из 2СС в 10СС:

     Y                
                     
                     
                     
     17                
     16                
     15                
     14                
     13                
     12                
     11                
     10                
     9                
     8                
     7                
     6                
     5                
     4                
     3                
     2                
     1                
     0 12
    3
    4
    5
    6
    7
    8
    9
    10
    11
    12
    13
    14
     15 16



    Document | slideum.

    com
    Transcript Document
    LOGO
    Методика
    подготовки к ГИА и ЕГЭ
    www.themegallery.com
    Системы
    счисления
    Безлюдная Ирина Сергеевна,
    учитель информатики МОУ-Лицея №2 г. Саратова
    LOGO
    Распределение заданий
    Системы
    счисления
    5 задания – 15,6%
    Код раздела
    1
    Код
    контролируемого
    элемента
    Элементы содержания,
    проверяемые на ЕГЭ
    1.4
    Системы счисления
    1.4.1
    Позиционные системы счисления
    1.4.2
    Двоичное представление
    информации
    www.themegallery.com
    LOGO Системы
    счисления
    Задания базового уровня сложности,
    время выполнения 1 минута.
    А5
    А1
    А9
    Задания повышенного уровня сложности,
    время выполнения 2 минуты.
    В7
    В11
    LOGO
    Структура подготовки
    Разбор заданий совместно (например, с
    использованием раздаточного
    материала Полякова)
    Подготовка учащимися учебных
    проектов
    Решение интересных задач
    Тестирование
    LOGO
    Что
    необходимо знать:
    Степень
    числа 2
    Десятичное
    значение
    20
    1
    21
    2
    22
    4
    23
    8
    24
    16
    25
    32
    26
    64
    27
    128
    28
    256
    29
    512
    210
    1024
    www. themegallery.com
    LOGO
    Что
    необходимо знать:
    Десятичное число
    Двоичное число
    1
    1
    2
    10
    3
    11
    4
    100
    5
    101
    6
    110
    7
    111
    8
    1000
    9
    1001
    10
    1010
    11
    1011
    12
    1100
    13
    1101
    14
    1110
    15
    1111
    LOGO
    Двоичная - восьмеричная
    2-ная
    000
    0
    8-ная
    001
    1
    010
    2
    011
    3
    100
    4
    101
    5
    110
    6
    111
    7
    Двоичная - шестнадцатеричная
    2-ная
    16-ная
    0000
    0001
    0010
    0011
    0100
    0101
    0110
    0111
    1000
    1001
    1010
    1011
    1100
    1101
    1110
    1111
    0
    1
    2
    3
    4
    5
    6
    7
    8
    9
    А
    В
    С
    D
    E
    F
    www.themegallery.com
    LOGO
    Алгоритм перевода из любой
    системы счисления в 10-ную
    Над каждой цифрой числа надписываем
    степени;
    Умножаем цифру на основание в степени;
    Всё складываем.
    Пример:
    4 3 2 1 -1
    1010,12 =1·24 + 1·22 + 1·2-1 =16+4+0,5=20,510
    www.themegallery.com
    LOGO
    Алгоритм перевода из 10-ной
    системы счисления в любую
    Целую часть делим на основание, пока
    делится нацело, остатки от деления
    переписываем в обратном порядке.
    Дробную часть умножаем на основание, пока
    не достигнем нулевой дробной части или
    заданной точности.
    www.themegallery.com
    Системы
    LOGO
    счисления
    А1.  (Диагностическая работа №1 от 14.12.2012)
    3 вариант.
    Для каждого из перечисленных ниже десятичных
    чисел построили двоичную запись. Укажите число,
    двоичная запись которого содержит наибольшее
    количество единиц.
    1) 13
    2)14
    3)15
    4)16
    4 вариант.
    Для каждого из перечисленных ниже десятичных
    чисел построили двоичную запись. Укажите число,
    двоичная запись которого содержит наибольшее
    количество значащих нулей.
    1)1
    2)7
    3)11
    4)15
    Системы счисления
    LOGO
    Для решения таких задач нужно помнить, что:
    четные числа оканчиваются на 0, нечетные – на 1;
    числа вида 2k записываются в двоичной системе как
    единица и k нулей, например: 16 = 24 = 100002
    числа вида 2k-1 записываются в двоичной системе k
    единиц,
    например: 15 = 24-1 = 11112
    если известна двоичная запись числа N, то двоичную
    запись числа 2·N можно легко получить, приписав в
    конец ноль, например:
    15 = 11112, 30 = 111102,
    60 = 1111002
    Системы
    LOGO
    счисления
    А1. (Диагностическая работа №1 от 14.12.2012)
    3 вариант.
    Для каждого из перечисленных ниже десятичных
    чисел построили двоичную запись.  Укажите число,
    двоичная запись которого содержит наибольшее
    количество единиц.
    1) 13
    2)14
    3)15
    4)16
    1510=11112, 1610=100002
    4 вариант.
    Для каждого из перечисленных ниже десятичных
    чисел построили двоичную запись. Укажите число,
    двоичная запись которого содержит наибольшее
    количество значащих нулей.
    1)1
    2)7
    3)11
    4)15
    1510=11112, 1110=10112
    Системы
    LOGO
    счисления
    А5. (Диагностическая работа №1 от 14.12.2012)
    3 вариант.
    Автомат получает на вход четырёхзначное восьмеричное число. По
    этому числу строится новое число по следующим правилам.
    1. Складываются первая и вторая, а также третья и четвёртая
    цифры.
    2. Полученные два числа записываются друг за другом в порядке
    возрастания (без разделителей).
    Пример. Исходное число: 4531. Суммы: 4+5 = 11; 3+1 = 4.
    Результат: 411.
    Определите, какое из следующих чисел может быть результатом
    работы автомата:
    3 вариант.
    1) 117
    2)1213
    3)1511
    4)1517
    4 вариант.
    1) 811
    2)717
    3)1511
    4)1214
    Системы
    LOGO
    счисления
    Решение:
    по условию цифры числа меньше или равны 7 (так
    как система восьмеричная), поэтому при сложении двух
    таких чисел может получиться сумма от 0 до 1410 = 168
     значит цифры 8 в записи числа быть не может, и
    каждая из двух сумм находится в интервале 0. .168
    по условию цифры записаны в порядке возрастания
    Вариант 3
    1)117
    2)1213
    3)1511
    4)1517
    Вариант 4
    1) 811
    2)717
    3)1511
    4)1214
    Системы
    LOGO
    счисления
    А9. (Диагностическая работа №1 от 14.12.2012)
    3 вариант.
    Для кодирования некоторой последовательности,
    состоящей из букв А, Б, В, Г и Д, решили использовать
    неравномерный
    двоичный
    код,
    позволяющий
    однозначно
    декодировать
    двоичную
    последовательность, появляющуюся на приёмной
    стороне канала связи. Для букв использовали
    кодовые слова: А–111, Б–110, В–101, Г– 100. Укажите,
    каким кодовым словом должна быть закодирована
    буква Д. Код должен удовлетворять свойству
    однозначного
    декодирования.
    Если
    можно
    использовать несколько кодовых слов, укажите
    кратчайшее из них.
    1) 1
    2) 0
    3)01
    4) 10
    Системы
    LOGO
    счисления
    Нужно знать, что:
    сообщение можно однозначно декодировать с
    начала, если выполняется условие Фано: никакое
    кодовое слово не является началом другого
    кодового слова;
    закодированное сообщение можно однозначно
    декодировать с конца, если выполняется обратное
    условие Фано: никакое кодовое слово не является
    окончанием другого кодового слова;
    условие Фано – это достаточное, но не
    необходимое условие однозначного декодирования. 
    если можно использовать несколько кодовых слов,
    укажите кратчайшее из них.
    Системы
    LOGO
    счисления
    А9. (Диагностическая работа №1 от 14.12.2012)
    3 вариант.
    Для кодирования некоторой последовательности, состоящей
    из букв А, Б, В, Г и Д, решили использовать неравномерный
    двоичный код, позволяющий однозначно декодировать
    двоичную последовательность, появляющуюся на приёмной
    стороне канала связи. Для букв использовали кодовые слова:
    А–111, Б–110, В–101, Г– 100. Укажите, каким кодовым словом
    должна быть закодирована буква Д. Код должен
    удовлетворять свойству однозначного декодирования. Если
    можно использовать несколько кодовых слов, укажите
    кратчайшее из них.
    1) 1
    2) 0
    3)01
    4) 10
    Системы
    LOGO
    счисления
    В7. (Диагностическая работа №1 от 14.12.2012)
    3 вариант.
    Десятичное число 59 в некоторой системе счисления
    записывается как 214. Определите основание этой
    системы счисления.
    4 вариант.
    Десятичное число 57 в некоторой системе счисления
    записывается как 212. Определите основание этой
    системы счисления. 
    Системы
    LOGO
    счисления
    Нужно знать:
    Для того, чтобы перевести число из системы счисления
    с основанием N в десятичную систему, нужно
    умножить значение каждой цифры на N в степени,
    равной ее разряду:
    Например,
    1 2 3 4 5N = 1·N4 + 2·N3 + 3·N2 + 4·N1 + 5·N0
    N0 = 1
    Тогда:
    3 вариант
    214N=5910
    2N2+N+4=59
    Решая квадратное уравнение
    получаем, что N=5
    4 вариант
    212N=5710
    2N2+N+24=57
    Решая квадратное уравнение
    получаем, что N=5
    Системы
    LOGO
    счисления
    В11. (Диагностическая работа №1 от 14.12.2012)
    3 вариант.
    В терминологии сетей TCP/IP маской сети называют двоичное число,
    которое показывает, какая часть IP-адреса узла сети относится к
    адресу сети, а какая – к адресу узла в этой сети. Адрес сети
    получается в результате применения поразрядной конъюнкции к
    заданному адресу узла и его маске. По заданным IP-адресу узла
    сети и маске определите адрес сети:
    IP-адрес: 224.32.249.137
    Маска: 255.255.240.0
    При записи ответа выберите из приведенных в таблице чисел 4
    фрагмента четыре элемента IP-адреса и запишите в нужном порядке
    соответствующие им буквы без точек. 
    A
    B
    C
    D
    E
    F
    255 249 240 224 137 32
    G
    8
    H
    0
    Системы
    LOGO
    счисления
    Нужно знать,
    •что каждая часть в IP-адресе (и в маске) – восьмибитное двоичное
    число, то есть десятичное число от 0 до 255
    •Так как 255 = 111111112, все части IP-адреса узла, для которых
    маска равна 255, входят в IP-адрес сети без изменений
    •Так как 0 = 000000002, все части IP-адреса узла, для которых маска
    равна 0, в IP-адресе сети заменяются нулями
    •таким образом, переведем в двоичную систему третью часть IPадреса и маски
    249 = 1111 10012
    240 = 1111 00002
    •выполним между этими числами поразрядную конъюнкцию –
    логическую операцию «И», получим число
    1111 00002 = 240
    •таким образом, полный адрес сети – 224.32.240.0
    •по таблице находим ответ: DFCH (D=224, F=32, C=240, H=0)
    Системы
    LOGO
    счисления
    В11. (Диагностическая работа №1 от 14.12.2012)
    4 вариант.
    В терминологии сетей TCP/IP маской сети называют двоичное число,
    которое показывает, какая часть IP-адреса узла сети относится к
    адресу сети, а какая – к адресу узла в этой сети.  Адрес сети
    получается в результате применения поразрядной конъюнкции к
    заданному адресу узла и его маске. По заданным IP-адресу узла
    сети и маске определите адрес сети:
    IP-адрес: 224.23.251.133
    Маска: 255.255.240.0
    При записи ответа выберите из приведенных в таблице чисел 4
    фрагмента четыре элемента IP-адреса и запишите в нужном порядке
    соответствующие им буквы без точек.
    A
    B
    C
    D
    E
    F
    255 240 252 224 133 23
    Ответ: DFBH
    G
    8
    H
    0
    Системы
    LOGO
    счисления
    B7. (Демо-версия ЕГЭ в компьютерной форме).
    Сколько единиц содержится в двоичной записи
    результата выражения:
    (2·108)2010 – 42011 + 22012?
    Системы
    LOGO
    счисления
    1. Переведем 108 в десятичную систему: 108 = 810
    2. 2*8 = 24
    3. (24)2010 = 28040, в двоичной системе это число представляет собой
    10000...0000
    8040
    4. 42011 = 24022, в двоичной системе это число представляет собой
    10000...0000
    4022
    5. Вычитаем второе число из первого по правилам вычитания в двоичной
    системе, получаем 11111..1111000..000
    (8040-4022=4018)
    6. 22012 в двоичной системе это число представляет собой
    10000. ..0000
    2012
    6)Складываем 4018+1= 4019 единиц.
    Системы
    LOGO
    счисления
    Аналогичная задача.
    Найти сумму восьмеричных чисел 178 +1708
    +17008 +...+17000008, перевести в 16-ую систему
    счисления.
    Найдите в записи числа, равного этой сумме,
    третью цифру слева.
    Системы
    LOGO
    счисления
    1. Выполним прямое сложение восьмеричных чисел:
    178 + 1708 = 2078
    178 + 1708 + 17008 = 21078
    178 + 1708 + 17008 + 170008 = 211078
    178 + 1708 + 17008 + 170008 + 1700008 = 2111078
    178+ 1708 + 17008 + 170008 + 1700008 + 17000008 = 21111078
    2. Переведем последнюю сумму через триады в двоичный
    код:
    100010010010010001112
    3. Переведем двоичное число в шестнадцатеричную систему
    счисления (разобьем на тетрады):
    100010010010010001112
    8
    9
    2
    4
    7
    4. Третья цифра слева: 2.
    Системы счисления
    LOGO
    

    Перевод дробных чисел из десятичной системы счисления в другую

    2. Перевод дробных чисел из десятичной системы счисления в другую.

    Правилоперевода дробных чисел из десятичной системы счисления в систему с основанием q:

    1. Последовательно выполнять умножение исходного числа и получаемых дробные части на q до тех пор, пока дробная часть не станет равна нулю или не достигнем требуемую точность.

    2. Полученные при таком умножении целые части — числа в системе счисления q – записать в прямом порядке (сверху вниз).

    Пример1. Перевести 0,562510 в двоичную систему счисления. А10→А2

    Решение:

    Ответ: 0,562510=0,10012

    Пример2. Перевести 0,562510 восьмеричную систему счисления. А10→А8

    Решение:

    Ответ: 0,562510=0,528

    Пример3. Перевести 0,66510 в двоичную систему счисления. А10→А2

    Р

    0, 665

    * 2

    1 330

    * 2

    0 660

    * 2

    0 320

    * 2

    0 640

    * 2

    1 280

    …………. .

    * 2

    0 5000

    * 2

    1 0000

    ешение:


    Процесс умножения может продолжаться до бесконечности. Тогда его прерывают на некотором шаге, когда считают, что получена требуемая точность представления числа


    Ответ: 0,66510=0,100012

    Переведите числа из десятичной системы счисления в другую.

    а) 0, 6562510А16

    б) 0,710А2 с точностью до 4 знаков после запятой

    в) 0,412510А8  с точностью до 6 знаков

    3. Перевод произвольных чисел из десятичной системы счисления в другую.

    Перевод произвольных чисел, то есть чисел, содержащих целую и дробную части, осуществляют в два этапа. Отдельно переводится целая часть, отдельно – дробная. В итоговой записи полученного числа целая часть отделяется от дробной запятой.

    Пример1. Перевести 26,2510 в двоичную систему счисления. А10→А2

    Решение:

    переводим целую часть переводим дробную часть



    Ответ: 26,2510=11010,012

    Пример2. Перевести 123,562510 в двоичную систему счисления. А10→А8

    Решение:

    переводим целую часть переводим дробную часть



    Ответ: 123,562510=173,448

    Перевести из десятичной системы счисления следующие числа:

    а) 173,562510А2

    б) 404,6562510А16

    в) 125,2510А8

    4. Перевод чисел из любой системы счисления в десятичную.

    ПравилоДля того чтобы число из любой системы счисления перевести в десятичную систему счисления, необходимо его представить в развернутом виде и произвести вычисления.

    Пример1. Перевести число 1101102 из двоичной системы счисления в десятичную.

    Решение:

    5 4 3 2 1 0

    1 1 0 1 1 0 2 = 1*25 + 1*24 + 0*23+1*22+1*21+0*20 =32+16+4+2=5410

    Ответ: 1101102 = 5410

    Пример2. Перевести число 101,012 из двоичной системы счисления в десятичную.

    Решение:

    2 1 0 -1 -2

    1 0 1,0 1 2 = 1*22 + 0*21 + 1*20+0*2-1+1*2-2 =4+0+1+0+0,25=5,2510

    Ответ: 101,012 = 5,2510

    Пример3. Перевести число 1221003 из троичной системы счисления в десятичную.

    Решение:

    4 3 2 1 0

    1 2 2 0 1 3=1*34 + 2*33 + 2*32 + 0*31 + 1*30 = 81+54+18+1 = 15410

    Ответ: 122013 = 15410

    Пример4. Перевести число 1637 из семеричной системы счисления в десятичную.

    Решение: 1637 = 1*72 + 6*71 + 3*70 = 49+42+3= 9410.

    Ответ: 1637 = 9410.

    Пример5. Перевести число 234,68 из восьмеричной системы в десятичную:

    Решение:

    2 1 0 -1

    2 3 4, 68 = 2*82 +3*81 + 4*80 +6*8-1= 2*64+3*8+4+6*0,125= 128+24+4+0,75 =156,7510

    Ответ: 234,68 = 156,7510.

    Пример6. Перевести число 2Е16 в десятичную систему счисления.

    Решение:

    2 1

    2 Е16 = 2*161 +14*160 = 32 +14 = 4610.

    Ответ: 2Е16 = 4610.

    Перевести из различных систем счисления в десятичную:

    а) 1111001112 г) 367,28

    б) 1001110,112 в) АВ2Е,816

    5. Перевод чисел из двоичной системы счисления в восьмеричную и шестнадцатеричную системы счисления.

    Перевод целых чисел.

    ПравилоЧтобы перевести целое двоичное число в восьмеричную (8=23) систему счисления необходимо:

    • разбить данное число справа налево на группы по 3 цифры в каждой;

    • рассмотреть каждую группу и записать ее соответствующей цифрой восьмеричной системы счисления.

    Пример1. Перевести число 111010102 в восьмеричную систему счисления.

    Решение:

    11101010

    3 5 2

    Ответ: 111010102 = 3528

    Пример2. Перевести число 111100000101102 в восьмеричную систему счисления.

    Решение:

    111110000010110

    7 6 0 2 6

    Ответ: 111100000101102= 760268

    ПравилоЧтобы перевести целое двоичное число в шестнадцатеричную (16=24) систему счисления необходимо:

    • разбить данное число справа налево на группы по 4 цифры в каждой;

    • рассмотреть каждую группу и записать ее соответствующей цифрой шестнадцатеричной системы счисления.

    Пример3. Перевести число 111000102 в шестнадцатеричную систему счисления.

    Решение:

    11100010

    Е 2

    Ответ: 111000102 = Е216

    Пример4. Перевести число 111100000101102 в шестнадцатеричную систему счисления.

    Решение:

    11110000010110

    3 С 1 6

    Ответ: 111100000101102= 3С1616

    Перевод дробных чисел.

    ПравилоЧтобы перевести дробное двоичное число в восьмеричную (шестнадцатеричную) систему счисления необходимо:

    • разбить данное число, начиная от запятой влево целую часть и вправо дробную часть на группы по 3 (4) цифры в каждой;

    • рассмотреть каждую группу и записать ее соответствующей цифрой восьмеричной (шестнадцатеричной)системы счисления.

    Пример5. Перевести число 0,101100001112 в шестнадцатеричную систему счисления.

    Решение:

    0,10110000111

    В 0 7

    Ответ: 0,101100001112 = В0716

    Пример6. Перевести число 111100001,01112 в восьмеричную систему счисления.

    Решение:

    111100001,0111

    7 4 1 3 1

    Ответ: 111100001,01112= 741,318

    Пример7. Перевести число 11101001000,110100102 в шестнадцатеричную систему счисления.

    Решение:

    11101001000,11010010

    7 4 8 D 2


    Ответ: 11101001000,110100102 = 748,D216

    Перевести числа в восьмеричную и шестнадцатеричную системы счисления:

    а) 11010001010112

    б) 100000011,0001011102

    в) 10010111011101,111010112

    г) 111110000000111111111,0000011111000001111101012

    6. Перевод чисел из восьмеричной и шестнадцатеричной систем счисления в двоичную систему счисления.

    ПравилоДля того, чтобы восьмеричное (шестнадцатеричное) число перевести в двоичную систему счисления, необходимо каждую цифру этого числа заменить соответствующим числом, состоящим из 3 (4) цифр двоичной системы счисления.

    Пример1. Перевести число 5288 перевести в двоичную систему счисления.

    Решение:

    5 2 3

    101 010 011

    Ответ: 5288 = 1010100112

    Пример2. Перевести число 4ВА35,1С216 перевести в двоичную систему счисления.

    Решение:

    4 В А 3 5 , 1 С 2

    100 1011 101000110101 0001 1100 0010


    Ответ: 4ВА35,1С216 = 10010111010001101010001 110000102

    Перевести числа в двоичную систему счисления

    а) 6217,2518 в) 236548

    б) А4ВС10А,5Е16 г) АСЕ560В16

    1 Переведите число из римской системы счисления в десятичную:

    MCMLXXXIV = ____________10

    2 Переведите число в римскую систему счисления:

    1499 = _______________________

    3 Представьте число в развернутой форме:

    235428,210 = ____________________________________________

    122231014 = ____________________________________________

    4 Переведите числа из десятичной системы счисления в другие:

    5610 = _____________2

    5610 = _____________5

    5 Переведите числа в десятичную систему счисления:

    110110112 = __________________10

    12223 = ____________________10

    3.

    Арифметические операции в системах счисления.
    1. Сложение в двоичной системе счисления.

    Правило

    0+0 =0

    1+0=1

    0+1=1

    1+1=10

    Пример1. Сложить числа 1112 и 102.

    Решение: 111

    + 10

    1001

    Проверка: 1112 = 710, 102= 210, 10012 =910 7+2=9

    Ответ: 10012

    Пример2. Сложить числа 111112 и 1112

    Решение: 11111

    + 111

    100110

    Проверка: 111112=1*24+1*23+1*22+1*21+1*20=16+8+4+2+1=3110

    1112 = 710

    1001102=1*25+0+0+1*22+1*21+0=32+4+2=3810

    31+7=38

    Ответ: 1001102

    Пример3. Сложить числа 11112, 10112, 1112.

    Решение:

    1111

    + 1011

    111

    100001

    (Пояснение: 1+1+1=11, 1 пишем,1 в уме, 1+1+1=11 плюс 1 в уме равно 100, 0 пишем, 0 в следующий разряд,1- через разряд и т.д.)

    Ответ: 1000012

    Пример4. Выполните сложение 1111,1012+101,112.

    Решение:

    111,101 + 101,11

    1101,011

    (Пояснение: по правилам математики при сложение дробных чисел запятая записывается под запятой)

    Ответ: 1101,0112

    Выполните действия:

    1. 111110011012+11111112 3) 111,11012+101,00112

    2. 1010101112+1111102 4) 111,01010112+101011,11112

    2. Вычитание в двоичной системе счисления.

    Правило

    0-0=0

    1-0=1

    1-1=0

    0-1=1 (занимаем у

    старшего разряда)

    Пример1. Из числа 10012 вычесть

    число 1112.

    Решение: _ 1001

    111

    10

    Проверка: 10012 =9, 1112 = 7, 102 = 2, 9-7=2

    Ответ: 102

    Пример2. Из числа 1000012 вычесть число 1112

    Решение: _ 100001

    111

    11010

    Ответ: 110102

    Пример3. Выполнить действие 100101,012 – 111,1112

    Решение: _ 100101,010

    111,111

    11101,101

    Ответ: 11101,1012

    Выполните действия:

    1) 111110011012-11111112

    2) 1010101112-1111102

    3) 111,11012-101,00112

    4) 101011,11112 — 111,01010112

    3. Умножение в двоичной системе счисления.

    Умножение в двоичной системе счисления производится аналогично умножению в десятичной системе счисления.

    Пример1. Умножить число 1012 на 1102

    Решение: 101

    *110

    000

    + 101

    101 .

    11110 Ответ: 111102

    Пример2. Выполнить действие 1011,012 ∙ 111,112

    Решение: 1011,01

    * 111,11

    101101

    101101

    + 101101

    101101

    101101 ,

    1010111,0011

    Ответ: 1010111,00112

    Выполните действия:

    1) 111110011012-11111112

    2) 111,11012-101,00112

    4. Деление в двоичной системе счисления.

    Операция деления выполняется также как и в десятичной системе счисления.

    Пример1.Разделить число 1010001012 на число 11012.

    Решение:

    101000101 1101

    1101 11001

    1110

    1101

    1101

    1101

    0 Ответ: 110012

    Пример2.Выполните деление с точностью до 3 знаков после запятой 10012:112

    Решение:

    1011 11

    11 . 11,010

    101

    11

    100

    11

    101

    11

    10 Ответ: 11,0102

    Выполните действия:

    1. 1011110011012:1101012

    2) Выполните деление с точностью до 4 знаков после запятой 10012:1012

    5. Сложение и вычитание в восьмеричной системе счисления.

    Используя таблицу и привычные правила сложения, совсем не трудно складывать и вычитать числа в восьмеричной системе счисления

    Правило

    Пример1. Вычислите 6348+2758

    Решение:

    634

    + 275

    1131 Ответ: 11318

    Пример2. Вычислите 305,48+24,758

    Решение:

    305,4

    + 24,75

    332,35 Ответ: 332,358

    Пример3. Вычислите 6348-2758

    Решение:

    634

    275

    337

    Ответ: 3378

    Пояснение: Т. к. от 4 не отнять 5, то занимаем у следующего разряда (т.к. система восьмеричная то 1 разряд составляет 8 единиц). От 8 -5+4=7

    Аналогично, т.к. у тройки одну единицу заняли, то необходимо от 2 отнять 7, поэтому, заняв у следующего разряда, получаем 8-7+2=3 и т.д.

    Пример4. Вычислите 305,48-24,758

    Решение:

    305,40

    24,75

    260,43 Ответ: 260,438

    Выполните действия:

    1. 560378+555728

    2. 536,2418+5673,668

    3. 50238— 44448

    4. 56,328-37,5678

    6. Умножение в восьмеричной системе счисления.

    Используя правило умножения и сложения восьмеричных чисел не трудно и перемножать данные числа

    Правило

    Пример. Вычислите 638∙27,58

    Решение:

    27,5

    63

    1067

    2156 ,

    2264,7

    Ответ: 2264,78

    Выполните умножение чисел:

    1. 560378∙555728

    2. 536,2418∙5673,668

    7. Сложение и вычитание в шестнадцатеричной системе счисления.

    Сложение и вычитание осуществляется аналогично таким же действиям в восьмеричной системе счисления

    Правило

    Пример. Вычислите

    E5F616+A0716

    E5F6

    A07

    EFFD

    Ответ: EFFD16

    Выполните действия:

    1. 5BE116+70EF3164) 7E1F3165BE16

    2. EB,5A16+7C,B7416 5) ADDC,1E16 — 789,B516

    8. Умножение в шестнадцатеричной системе счисления.

    Правило

    Пример

    FFA,3

    * D,E

    DFAEA

    CFB47 /

    DDAF,5A

    Выполните действия:

    1. 3ED16∙A0516

    2. 5C2,5A16∙3D,9EF16

    4.

    Нестандартные задачи

    На Новый год на ёлке висело 32 игрушки и 11 конфет. Всего 103 предмета. В какой системе счисления записаны числа?

    Решение:

    32р = 3*р+2

    11р = 1*р+1

    103р = 1*р2 +0*р+3

    32р + 11р = 103р

    3*р+2 + 1*р+1 = 1*р2 +0*р+3

    4р+3 =р2 +3

    р2 – 4р=0

    р(р-4)=0

    р1 = 0; р2 = 4.

    Так как основание системы счисления не может быть меньше 2, то ответом является число 4.

    Ответ: В данном примере использована 4-ичная система счисления.

    Вычислите 101112 — 5116 : 338, записав результат в двоичной системе счисления.

    Решение:

    Привычнее всего производить расчеты в десятичной системе счисления, поэтому сначала переведем все числа в десятичную систему и затем произведем соответствующие вычисления.

    101112 = 2310

    5116 = 8110

    338 = 2710

    8110 : 2710 = 310

    2310310 = 2010

    2010 = 101002

    Ответ: 101002

    1 Переведите число из двоичной системы счисления в десятичную:

    110110112= ____________10

    2 Переведите числа из двоичной системы счисления в соответствующие:

    11011110112 = ________________________________8

    11011110112 = ________________________________16

    3 Переведите числа из соответствующих систем счисления в двоичную:

    355728 = _________________________2

    А517ВЕ16 = _______________________2

    4 Выполните действия:

    1101112 + 111102 = ______________________2

    1101112 — 111102 = _______________________2

    1101112 * 111102 = _______________________2

    5 Вычислить:

    A1CA16:1658 – (1000101002:1210 + 100000112)= _____________2

    Индивидуальная работа учащегося

    Выполнить следующие задания:

    • Переведите двоичные числа в восьмеричную систему счисления: 101011011; 100010,011101; 0,000110101

    • Переведите двоичные числа в шестнадцатеричную систему счисления: 111111; 100000111,001110; 0,011011011

    • Переведите восьмеричные числа в двоичную систему счисления: 276; 0,635; 25,024

    • Переведите шестнадцатеричные числа в двоичную систему счисления 1А2С7; 0,3С1; F4A,C1C

    • Переведите числа из шестнадцатеричной системы счисления в восьме­ричную: А54; 21E,7F; 0,FD

    • Переведите числа из восьмеричной системы счисления в шестнадцате­ричную: 344; 0,7612; 333,222

    • Выполните сложение:

    1001001+10101

    101101 + 1101101

    1110101 + 1001101+111101

    11000,11+11010,11

    10001000-1110011

    11010110-10101110

    1111001-1010111

    1101100-10110110

    111010*10010

    11100*10110

    11010*10110

    100001*111,11

    1000000:1110

    10111001101:110101

    11101001001:111101

    100111:1100

    345+502; 46,2*64,4; 312*226; 502,23+612,15

    3ЕА5С+235ВА; 35DB*7A2

    ТВОРЧЕСКИЙ УРОВЕНЬ:

    **0*01**1 + 10111*10**=100*1*00010

    ***0**00 — 11*11*11=1101*1

    1*01 · 1**=101101

    • Подсчитайте сумму всех троичных чисел в диапазоне от 103 до 1123, включая границы диа­пазона. Ответ запишите в двоичной системе счисления.

    • Какое число необходимо прибавить к числу 1001111011112 чтобы получилось сокращенное английское название дисковода для мягких магнитных дисков. Ответ записать в восьмеричной системе счисления.

    Литература

    1. Андреева Е., Фалина И. Системы счисления и компьютерная арифметика. Изд. 2-е, — М.: Лаборатория Базовых Знаний, 2000.

    2. Угринович Н.Д. Информатика и информационные технологии. Учебник для 10-11 классов. – М.: Бином. Лаборатория Знаний, 2002.

    преобразовать двоичное число 100110 в десятичное

    100110 двоичное в десятичное — двоичное в десятичное Пошаговый преобразователь / калькулятор базы чисел.

    Преобразование из / в десятичное, шестнадцатеричное, восьмеричное и двоичное. Калькулятор преобразования двоичной базы. Здесь вы можете найти ответы на такие вопросы, как: преобразовать двоичное число 100110 в десятичное или преобразовать двоичное в десятичное.

    Таблица десятичных, двоичных, шестнадцатеричных и восьмеричных диаграмм

    90 020
    Дек Hex Октябрь Корзина
    0 0 0 0
    1 1 1 1
    2 2 2 10
    3 3 3 11
    4 4 4 100
    5 5 5 101
    6 6 6 110
    7 7 7 111
    8 8 10 1000
    9 9 11 1001
    10 A 12 1010
    11 B 13 1011
    12 C 14 1100
    13 D 15 1101
    14 E 16 1110
    15 F 17 1111
    Дек Hex Окт Корзина
    16 10 20 10000
    17 11 21 10001
    18 12 22 10010
    19 13 23 10011
    20 14 24 10100
    21 15 25 10101
    22 16 26 10110
    23 17 27 10111
    24 18 30 11000
    25 19 31 11001
    26 1A 32 11010
    27 9002 3 1B 33 11011
    28 1C 34 11100
    29 1D 35 11101
    30 1E 36 11110
    31 1 этаж 37 11111
    90 022 43
    Dec Hex Oct Bin
    32 20 40 100000
    33 21 41 100001
    34 22 42 100010
    35 23 43 100011
    36 24 44 100100
    37 25 45 100101
    38 26 46 100110
    39 27 47 100111
    40 28 50 101000
    41 29 51 101001
    42 2A 52 101010
    2B 53 101011
    44 2C 54 101100
    45 2D 55 101101
    46 2E 56 101110
    47 2F 57 101111
    90 022 59
    Dec Hex Oct Корзина
    48 30 60 110000
    49 31 61 110001
    50 32 62 110010
    51 33 63 110011
    52 34 64 110100
    53 35 65 110101
    54 36 66 110110
    55 37 67 110111
    56 38 70 111000
    57 39 71 111001
    58 3A 72 111010
    3B 73 111011
    60 3C 74 111100
    61 3D 75 111101
    62 3E 76 111110
    63 3F 77 111111
    Dec Hex Oct Корзина
    64 40 100 1000000
    65 41 101 1000001
    66 42 102 1000010
    67 43 103 1000011
    68 44 104 1000100
    69 45 105 1000101
    70 46 106 1000110
    71 47 107 1000111
    72 48 110 1001000
    73 49 111 1001001
    74 4A 112 1001010 9 0023
    75 4B 113 1001011
    76 4C 114 1001100
    77 4D 115 1001101
    78 4 116 1001110
    79 4F 117 1001111
    Dec Hex Oct Корзина
    80 50 120 1010000
    81 51 121 1010001
    82 52 122 1010010
    83 53 123 1010011
    84 54 124 1010100
    85 55 125 1010101
    86 56 126 1010110
    87 57 127 1010111
    88 58 130 1011000
    89 59131 1011001
    90 5A132 1011010 9 0023
    91 5B 133 1011011
    92 5C 134 1011100
    93 5D 135 1011101
    94 5 136 1011110
    95 5F 137 1011111
    Dec Hex Oct Корзина
    96 60 140 1100000
    97 61 141 1100001
    98 62 142 1100010
    99 63 143 1100011
    100 64 144 1100100
    101 65 145 1100101
    102 66 146 1100110
    103 67 147 1100111
    104 68 150 1101000
    105 69 151 1101001
    106 6A 152 11 01010
    107 6B 153 1101011
    108 6C 154 1101100
    109 6D 155 1101101
    6E 156 1101110
    111 6F 157 1101111
    9002 2 1111010 126
    Dec Hex Oct Корзина
    112 70 160 1110000
    113 71 161 1110001
    114 72 162 1110010
    115 73 163 1110011
    116 74 164 1110100
    117 75 165 1110101
    118 76 166 1110110
    119 77 167 1110111
    120 78 170 1111000
    121 79 171 1111001
    122 7A 172
    123 7B 173 1111011
    124 7C 174 1111100
    125 7D 175 1111101
    7E 176 1111110
    127 7F 177 1111111
    Dec Hex Oct Корзина
    128 80 200 10000000
    129 81 201 10000001
    130 82 202 10000010
    131 83 203 10000011
    132 84 204 10000100
    133 85 205 10000101
    134 86 206 10000110
    135 87 207 10000111
    136 88 210 10001000
    137 89 211 10001001
    138 8A 212 9 0023 10001010
    139 8B213 10001011
    140 8C 214 10001100
    141 8D215 100011011 900 8E 216 10001110
    143 8F 217 10001111
    Dec Hex Oct Корзина
    144 90 220 10010000
    145 91 221 10010001
    146 92 222 10010010
    147 93 223 10010011
    148 94 224 10010100
    149 95 225 100201011 150 96 226 10010110
    151 97 227 10010111
    152 98 230 10011000
    153 99 153 99 10011001
    154 9A 232 9 0023 10011010
    155 9B 233 10011011
    156 9C 234 10011100
    157 9D 235 1002011 9E 236 10011110
    159 9F 237 10011111
    Dec Hex Oct Bin
    160 A0 240 10100000
    161 A1 241 10100001
    162 A2 900 242 10100010
    163 A3 243 10100011
    164 A4 244 10100100
    165 A5 245 101001011
    166 A6 246 10100110
    167 A7 247 10100111
    168 A8 250 10101000
    169
    169 10101001
    170 AA 252 9 0023 10101010
    171 AB 253 10101011
    172 AC 254 10101100
    173 AD 255 1010201 AE 256 10101110
    175 AF 257 10101111
    Dec Hex Oct Bin
    176 B0 260 10110000
    177 B1261 10110001
    178 B2 262 10110010
    179 B3263 10110011
    180 B4 264 10110100
    181 B5 265
    182 B6266 10110110
    183 B7 267 10110111
    184 B8 270 10111000
    185 10111000
    185 10111001
    186 BA 272 9 0023 10111010
    187 BB 273 10111011
    188 BC 274 10111100
    189 BD 275 1011110 BE 276 10111110
    191 BF 277 10111111
    Dec Hex Oct Корзина
    192 C0300 11000000
    193 C1 301 11000001
    194 C2 302 11000010
    195 C3 303 11000011
    196 C4 304 11000100
    197 C5 305 112011 305 112011 198 C6306 11000110
    199 C7 307 11000111
    200 C8 310 11001000
    201 C9 11001001
    202 CA 312 9 0023 11001010
    203 CB313 11001011
    204 CC314 11001100
    205 CD315 11002011 CE 316 11001110
    207 CF 317 11001111
    011021031041051071081011

    27

    60 61 62 63 64 65 66 67 68 69 6A 6B 6C 6D 6E 6F

    `abcdefghijklmno

    шестнадцатеричный символ ASCII 48 49 50 51 52 53 54 55 56 57 58 59 60 61 62 63

    30 31 32 33 34 35 36 37 38 39 3A 3B 3C 3D 3E 3F

    0 1 2 3 4 5 6 7 8 9:; ?

    Шестнадцатеричный символ ASCII 112 113 114 115 116 117 118 119 120 121 122 123 124 125 126 127

    70 71 72 73 74 75 76 77 78 79 7A 7B 7C 7D 7E 7F

    p q r s t u v w x y z {| } ~

    Каждый символ ASCII показан как в шестнадцатеричном представлении, так и в представлении с основанием 10.Предположим, мы хотим узнать, как символ вопросительного знака хранится внутри компьютера. Просматривая таблицу на предмет вопросительного знака, мы замечаем, что его шестнадцатеричное значение — 3F. Преобразуя это шестнадцатеричное значение в двоичное, мы заключаем, что вопросительный знак сохраняется как 00111111. Пример Как буква t сохраняется в памяти? Решение:

    Предположим, вы используете способности супергероя, чтобы заглянуть в ячейку памяти, и вы видите: 01110100. Что означает количество, хранящееся в этом месте? Если вы конвертируете это двоичное значение в целое число, вы сделаете вывод, что целое число 116 хранится в этом месте.С другой стороны, глядя на таблицу ASCII, можно сделать вывод, что символ t хранится в этом месте. Что правильно? Значение каждого фрагмента данных, хранящегося в компьютере, определяется программистом. Например, предположим, что программист использует переменную с именем midcode для хранения значения. Если программист предполагает, что это значение будет целым числом, равным 116, он может включить строку кода: int midcode = 116; С другой стороны, если программист предполагает, что значение будет символом t, он вместо этого будет использовать эту строку кода: char midcode = ‘t’; В обоих случаях данные сохраняются как 01110100.Компьютер интерпретирует данные в соответствии с инструкциями, предоставленными программистом. Если программист проинструктировал компьютер интерпретировать промежуточный код как целое число, то битовая строка 01110100 будет интерпретироваться как целое число 116. С другой стороны, если программист проинструктировал компьютер интерпретировать промежуточный код как символ, тогда битовая строка 01110100 будет интерпретироваться как буква t. 1.10. Десятичное число с двоичным кодом. Завершим кратким обзором различных способов преобразования двоичного числа в десятичное.В некоторых практических приложениях мы будем использовать цифровую логическую схему для управления числовым дисплеем, где каждый отдельный дисплейный блок отображает одну цифру. Так, например, у нас может быть номер 472 в нашей логической схеме, и мы хотели бы отобразить его на трех отдельных дисплеях (один для 4, один для 7 и один для 2). Работа с таким дисплеем упрощается за счет использования десятичного двоичного кода (BCD), где каждая отдельная цифра представлена ​​4-битным числом.Например, чтобы представить десятичное число 472 в двоично-десятичной системе счисления (BCD), мы преобразуем каждую цифру в четырехбитное двоичное число, независимо от других десятичных цифр. Таким образом, 472 равно 0100 0111 0010 в BCD.

    28

    Проблемы 1.1

    Преобразуйте следующие двоичные числа в десятичные: (a) (b) (c) (d)

    1.2

    10101 11001 100000 111111

    Преобразуйте следующие десятичные числа в двоичные: ( a) (b) (c) (d)

    1,3

    50100 128 500

    Выполните сложение следующих беззнаковых (т.е., не дополненные до двух) двоичные числа: (a) +

    1.4

    +

    1011 + 0011

    (c)

    1110011 + 0011110

    111100 001011

    (b)

    (c)

    101100 + 001100

    111001 + 011001

    Предположим, мы используем 6 бит для хранения целых чисел. Покажите следующие дополнения в двоичном формате: (a) (b) (c) (d)

    1,6

    (b)

    Предположим, что числа без знака хранятся в пяти битах. Выполните следующие добавления, указав, какие добавления вызывают переполнение.(a)

    1,5

    1111 1010

    10 + 5 32 + 31 1 + 31 21 + 21

    Выразите следующие числа как 8-битные двоичные числа с дополнением до двух: (a) (b) (c) (d) (e) (f) (g) (h)

    60-60 1-1 0 12-15-128

    29

    1,7

    Преобразуйте следующие шестибитовые числа с дополнением до двух в десятичное: ( a) (b) (c) (d)

    1,8

    Покажите каждое из следующих вычислений в 4-битной записи с дополнением до двух. В каждом случае преобразуйте результат (в двоичном формате) обратно в десятичное число.(a) (b) (c) (d)

    1,9

    –5 + 7 –3 + 3 6 4 2 3

    Предположим, мы используем шесть битов для представления двоичного числа с дополнением до двух. (а) (б) (в)

    1.10.

    010111 110111 011111 100000

    Какое наибольшее число может быть представлено? Какое наименьшее число можно представить? Сколько всего десятичных чисел можно представить?

    Предположим, мы используем шесть битов для представления двоичного числа с дополнением до двух. Выполните следующие добавления, указав, когда произойдет переполнение: (a)

    010101 001101

    (b)

    +

    010101 101111

    (d)

    +

    (c)

    1.11.

    +

    100000 110011

    +

    100110 111111

    Предположим, мы используем шесть битов для представления двоичного числа с дополнением до двух. Выполните следующие вычитания, указав, когда произойдет переполнение: (a)

    010101 001101

    (b)

    010101 101111

    (d)

    (c)

    — 1000003 110002

    72036854775807) в двоичное значение.Деление карандаша и бумаги, также известное как деление в столбик, является самым сложным из четырех арифметических алгоритмов. Введите положительные или отрицательные десятичные числа для делителя и делимого и вычислите ответ частного. Ускоренное преобразование двоичного числа в десятичное 1 Преобразование двоичного числа в десятичное без деления Сирил Бувье и Пол Циммерманн Аннотация. В этой статье представлены алгоритмы, которые преобразуют целые числа с множественной точностью или числа с плавающей запятой из системы счисления 2 в систему счисления 10 (или в любую систему счисления b> 2). Я действительно хочу поблагодарить издателя, который опубликовал это. Например, бинарная мантисса 1.1 с показателем -1 будет означать десятичное 0,75 (двоичное 1,1 == десятичное 1,5, а показатель -1 означает «делить на 2» точно так же, как десятичный показатель -1 означает «делить на 10»). Преобразование десятичных чисел в числа с плавающей запятой с использованием больших целых чисел. • «Плавающая точка»: двоичная (десятичная) точка имеет разную величину + «скользящее окно» точности с использованием понятия значащих цифр • Маленькие числа очень точны, много знаков после десятичной точки • Большие числа намного меньше, не все целые числа представимы • Но для этих случаев вам все равно все равно (на самом деле, более прямой способ использовать этот инструмент — ввести ‘2’ для ‘New Base’; это дает дробь 101111/1100, что эквивалентно нашей проблеме деления 1011.11/11.). Этот пример соответствует второму пункту моего заголовка «Другие случаи»: просто сдвиньте десятичную запятую так, чтобы деление было 10011000/1011. … Сложите результаты. Эта процедура называется длинным делением. Нет, потому что оно больше 10. Например, чтобы представить 0,25 в двоичной системе… Каждая красная цифра перечеркивается перед следующим умножением. Это не 7,22 или 15,95 цифр. Деление двоичных чисел проще деления десятичных чисел. Переместите десятичную точку в делителе и делимом.Помимо этих подзадач деления также требуются умножение и вычитание. (Есть также аналитические способы точно проверить ответ: прочтите мои статьи о методе вычитания, прямом методе и методе рядов.) Сначала введите первичное число (в двоичном формате; убедитесь, что оно действительное), затем введите вторичное число (также в двоичном формате) для расчета и нажмите «Рассчитать». «17 цифр» поможет вам, когда вы нашли свой путь. Если делитель или делимое отрицательное, вы можете убрать знаки и применить соответствующий знак к ответу в конце.Десятичное число равно сумме двоичных цифр (d n), умноженной на их степень 2 (2 n) :. Красные цифры — это переносы, которые происходят во время подшагов умножения (умножение выполняется так, как если бы делитель — большее число — по соглашению находился сверху). Это объясняется следующим образом. Опять же, это похоже на 11.1110. Превратите делитель (число, на которое вы делите) в целое число, сдвинув десятичную точку до упора вправо. Запишите номер. Я могу опубликовать результаты тестирования некоторых ваших программ.Теперь давайте преобразуем то же число обратно в двоичную форму с фиксированной точкой. Еще мне нравится изображение в «поездке туда и обратно кратчайших чисел». Большое спасибо! Деление на двоичное число 1 с последующими k нулями выполняется смещением точки на k шагов влево. Получите остаток от двоичной цифры. Затем мы должны записать 0, потому что 88 превращается в 83 ноль раз, умножить 88 на 0, вычесть 0 из 83 и уменьшить 1. Хотя двоичное деление проще, чем десятичное деление (потому что здесь нет угадывания и фактически нет умножения) , вы обнаружите, что всегда имея одно и то же число (делитель) в качестве вычитаемого, вы получите узор, который начнет вас завораживать; в этом море нулей и единиц легко потеряться.Но в отличие от других алгоритмов, здесь нет ограниченного набора «фактов», решающих все возможные подзадачи. Например, предположим, что вы хотите разделить 10,274 на 0,11. Затем замените a на латекс a-d * b и повторите процесс, чтобы найти следующую ненулевую цифру частного и так далее. Я мог бы выразить дробную часть как целочисленный остаток или в дробной форме. Да, потому что оно меньше или равно 101. Нет, потому что оно больше 1. Десятичная, двоичная, шестнадцатеричная и восьмеричная системы. Двоичное деление Хорошей новостью является то, что двоичное деление намного проще, чем десятичное.1111111 ÷ 101 Вычесть 4. Получите целое частное для следующей итерации. 1 ÷ 1 = 1 2. Как и другие арифметические алгоритмы, я описал алгоритм деления независимым от основания способом. это делает вычисление повторяющейся части более очевидным (захват цифр частного против остатка [включая выпадающий список]). 88 переходит в 83? Деление двоичных чисел проще деления десятичных чисел. Каждая позиция цифры в двоичном числе представляет собой степень 0 основания (2).Нет необходимости гадать, а затем проверять промежуточные частные; они равны либо 0, либо 1, и их легко определить на глаз. Как преобразовать двоичное в десятичное. Иногда для знака или других указаний используются специальные битовые комбинации (например, это очень ясно. Цифры частного q могут вычисляться последовательно слева направо, причем первая ненулевая цифра равна цифре наибольшего однозначного числа d Таким образом, двоичное деление намного проще, чем десятичное деление, если вы помните следующие правила деления.Давайте рассмотрим, как выполняется десятичное деление, чтобы мы могли подготовить почву для того, как выполняется деление в двоичной системе. Напишите код, реализующий двоичное деление в столбик. Добро пожаловать — спасибо за отзыв. это облегчило мне двоичное деление. Затем преобразуйте дробную часть в двоичную форму. Вы также можете проверить ответ с помощью моего двоичного калькулятора. Процесс преобразования десятичного числа в двоичное Начиная с младшего значащего бита, умножьте цифру на значение заполнителя. как разделить это двоичное число 100110/10.11 ????????? До сих пор мы имели дело с целыми числами. Если вы поиграете с двоичным делением, то увидите, что оно дает больше повторяющихся дробных чисел, чем десятичное. 1101 ÷ 101. Например, 2/5 = 0,4, но 10/101 = 0,0110. Вы также можете использовать этот инструмент для преобразования в обратном направлении, убедившись, что 3.916 преобразуется в 11.1110. Проблема кажется из-за целого размера. (Я оставлю это как упражнение, но ответ — 1011.11000001, что очень близко к 1011.11). Я мог бы выбрать проблему с однозначным делителем (который не потребовал бы никаких предположений, если вы знаете факты умножения), или с делителем целого числа, или с делителем с дробной частью, которая завершалась.Что мы можем сделать, так это приблизить частное к конечному количеству знаков, а затем проверить, что оно приближается к ожидаемому ответу. Я хотел бы прояснить это, если так. Вы так быстро отвечаете. Например, 10.16 — десятичное число с плавающей запятой. Если делитель имеет десятичную точку, сдвиньте десятичную точку вправо, пока делитель не станет целым числом, и сдвиньте делимое на такое же количество разрядов. При преобразовании десятичного числа в двоичное мы преобразуем число с основанием 10 в число с основанием 2, используя простые методы.Например, если 12 10 — десятичное число, то его эквивалентное двоичное число равно 1100 2 .. Здесь учащиеся могут научиться преобразовывать любое заданное десятичное число в его эквивалентную двоичную систему счисления. В этой системе счисления вы, возможно, узнали о различных типах. чисел, таких как; Вы можете проверить ответ несколькими способами. Карандашно-бумажный метод двоичного деления такой же, как карандашный и бумажный метод десятичного деления, за исключением того, что вместо этого используются двоичные числа. Решение этих подзадач деления требует оценки, предположений и проверки.Вам не нужна докторская степень. преобразовать в числа с плавающей запятой. Вы можете нажать «Очистить значения», чтобы сделать другое. Двоичные числа, двоичный код и двоичная логика. В дополнение к этим подзадачам деления, умножьте… Значение Пи (Π) до 50 знаков после запятой; Количество цифр перед десятичной точкой при делении двух чисел; Преобразование десятичного числа в шестнадцатеричное, включая отрицательные числа; Общее количество способов разместить X и Y в n местах так, чтобы никакие два X не находились вместе; Проверка на простоту суммы цифр в нечетных местах числа Остановите нажатие! Похоже, он хочет быть 11.1110, ответ мы получили с использованием двоичного деления. Однако есть два прямых метода преобразования десятичного числа в двоичное: выполнение краткого деления на два с остатком (для целой части), выполнение краткого умножения на два с результатом (для дробной части) и убывающая степень двойки и вычитание. . Вид обмана. Но, как оказалось, я ошибаюсь во всех отдельных случаях. Вы не сказали, с чем вам нужна помощь, но если вам просто нужны ответы, попробуйте мой двоичный калькулятор.Так как 9 * 88, 9 * 8 = 72, напишите 2 и перенесите 7; для 4 * 88, 4 * 8 = 32, поэтому напишите 2 и перенесите 3; так далее. . Вот пример такого преобразования с использованием дроби 0,1011. Вот оно, на случай, если вы еще не догадались: ваше объяснение очень ясное, однако меня озадачивают те числа, которые вы ставите над делителем, 4 3 3 7, которые вы зачеркиваете. Пластическая хирургия в Мичигане Цена, Сахарная ласка на продажу, Граница Тексты Бренди, Руководство по выбору размеров искры, Почему переходные элементы образуют сложные соединения, Есть ли у крокуса семена, Демография читателей журнала, Программа Health Quest Резиденция семейной медицины, Discount Flooring Луисвилл, штат Кентукки,

    двоичное деление с десятичной точкой 2020

    У вас недостаточно прав для чтения этого закона в это время

    У вас недостаточно прав для чтения этого закона в это время Логотип Public.Resource.OrgЛоготип представляет собой черно-белую линию улыбающегося тюленя с усами. Вокруг печати красная круглая полоса с белым шрифтом, в верхней половине которого написано «Печать одобрения», а в нижней половине — «Public.Resource.Org». На внешней стороне красной круглой марки находится круг. серебряная круглая полоса с зубчатыми краями, напоминающая печать из серебряной фольги.

    Public.Resource.Org

    Хилдсбург, Калифорния, 95448
    Соединенные Штаты Америки

    Этот документ в настоящее время недоступен для вас!

    Уважаемый гражданин:

    В настоящее время вам временно отказано в доступе к этому документу.

    Public Resource ведет судебный процесс за ваше право читать и говорить о законах. Для получения дополнительной информации см. Досье по рассматриваемому судебному делу:

    Американское общество испытаний и материалов (ASTM), Национальная ассоциация противопожарной защиты (NFPA), и Американское общество инженеров по отоплению, охлаждению и кондиционированию воздуха (ASHRAE) против Public.Resource.Org (Public Resource), DCD 1: 13-cv-01215, Объединенный окружной суд округа Колумбия [1]

    Ваш доступ к этому документу, который является законом Соединенных Штатов Америки, был временно отключен, пока мы боремся за ваше право читать и говорить о законах, по которым мы решаем управлять собой как демократическим обществом.

    Чтобы подать заявку на получение лицензии на чтение этого закона, ознакомьтесь с Сводом федеральных нормативных актов или применимыми законами и постановлениями штата. на имя и адрес продавца. Для получения дополнительной информации о указах правительства и ваших правах как гражданина в соответствии с нормами закона , пожалуйста, прочтите мое свидетельство перед Конгрессом Соединенных Штатов. Вы можете найти более подробную информацию о нашей деятельности на публичном ресурсе. в нашем реестре деятельности за 2015 год. [2] [3]

    Спасибо за интерес к чтению закона.Информированные граждане — это фундаментальное требование для работы нашей демократии.

    Leave A Comment

    Dec Hex Oct Bin
    208 D0320 11010000
    209 D1 321 11010001
    210 D 322 11010010
    211 D3 323 11010011
    212 D4 324 11010100
    213 D5 325
    214 D6 326 11010110
    215 D7 327 11010111
    216 D8 330 11011000
    217 D 11011001
    218 DA 332 9 0023 11011010
    219 DB 333 11011011
    220 DC 334 11011100
    221 DD 335 11011101
    DE 336 11011110
    223 DF 337 11011111
    Dec Hex Oct Bin
    224 E0 340 11100000
    225 E1341 11100001
    226 E0 342 11100010
    227 E3343 11100011
    228 E4 344 11100100
    229 E5 34510
    E5 34510 230 E6346 11100110
    231 E7 347 11100111
    232 E8 350 11101000
    233 E 11101001
    234 EA 352 9 0023 11101010
    235 EB353 11101011
    236 EC 354 11101100
    237 ED 355 237 355 1110201 EE 356 11101110
    239 EF 357 11101111
    Dec Hex Oct Корзина
    240 F0 360 11110000
    241 F1 361 11110001
    242 F2 362 11110010
    243 F3363 11110011
    244 F4 364 11110100
    245 F5 36510
    246 F6 366 11110110
    247 F7 367 11110111
    248 F8 370 11111000
    249 11111001
    250 FA 372 9 0023 11111010
    251 FB 373 11111011
    252 FC 374 11111100
    253 FD 375 11111 FE376 11111110
    255 FF 377 11111111

    Конвертер числовой базы

    Пожалуйста, дайте ссылку на эту страницу! Просто щелкните правой кнопкой мыши изображение выше, затем выберите копировать адрес ссылки и вставьте его в свой HTML.

    Преобразование базового числа отсчетов

    Заявление об ограничении ответственности

    Несмотря на то, что прилагаются все усилия для обеспечения точности информации, представленной на этом веб-сайте, ни этот веб-сайт, ни его авторы не несут ответственности за какие-либо ошибки или упущения или за результаты, полученные в результате использования этой информации. Вся информация на этом сайте предоставляется «как есть», без гарантии полноты, точности, своевременности или результатов, полученных в результате использования этой информации.

    двоичных чисел ▷ Таблица / список чисел от 0 до 100 в двоичном формате

    На сайте Convert Binary dot com вы можете найти числа от 0 до 100 в их двоичном кодовом представлении.

    Если вы хотите узнать двоичное представление любого десятичного числа длиной до 7 цифр, обратите внимание на преобразователь десятичного числа в двоичное.

    ДЕСЯТИЧНЫЕ ЧИСЛА В ДВОИЧНОМ УРОВНЕ

    3 9 0011 9 0011 84 22

    1111111

    22 900
    Десятичное Двоичное
    0 0
    1 1
    2
    11
    4 100
    5 101
    6 110
    7 111
    8 1000
    9 1001
    10 1010
    11 1011
    12 1100
    13 1101
    14 1110
    15 1111
    16 10000
    17 10001
    18 10010
    19 10011
    20 10100
    21 10101
    22 10110
    23 10111
    24 11000
    25 11001
    26 11010
    27 11011
    28 11100
    29 11101
    30 11110
    31 11111
    32 100000
    33 100001
    34 100010
    35 100011
    36 100100
    37 100101
    38 100110
    39 100111
    40 101000
    41 101001
    42 101010
    43 101011
    440022 101100
    45 101101
    46 101110
    47 101111
    48 110000
    49 110001
    50 110001
    50
    51 110011
    52 110100
    53 110101
    54 110110
    55 110111
    56 111000
    56 111000
    57 111001
    58 111010
    59 111011
    60 111100
    61 111101
    62 111110
    63 111111
    1000000
    65 1000001
    66 1000010
    67 1000011
    68 1000100
    69 1000101
    70 1000110 900
    71 1000111
    72 1001000
    73 1001001
    74 1001010
    75 1001011
    76 1001100
    77 1001101
    78 1001110
    79 1001111
    80 1010000
    81 1010001
    82 1010010
    83 1010011
    1010100
    85 1010101
    86 1010110
    87 1010111
    88 1011000
    89 1011001
    89 1011001
    1011010
    91 1011011
    92 1011100
    93 1011101
    94 1011110
    95 1011111
    97 110 0001
    98 1100010
    99 1100011
    100 1100100

    Также обратите внимание на двоичный алфавит!

    Вопросы и ответы о двоичных числах

    🔟 Как вы читаете двоичные числа?

    Чтобы читать двоичные числа и преобразовывать их в их десятичный эквивалент, у вас есть два варианта: вы можете использовать преобразователь двоичных чисел в десятичные числа в ConvertBinary. com, или вы можете сделать это вручную.

    Короче говоря, чтобы преобразовать двоичные числа в десятичные, вы должны умножить каждую двоичную цифру на два в степени ее разряда справа налево, а затем сложить все результаты вместе. При вычислении разряда самая правая цифра разряда имеет значение ноль.

    Так, например, если вы хотите преобразовать двоичное 1010 в десятичное, вы начинаете с крайнего правого 0.

    Давайте сделаем это с двоичным 1010:
    0 × 2 0 = 0
    1 × 2 1 = 2
    0 × 2 2 = 0
    1 × 2 3 = 8

    Добавьте 0 + 2 + 0 + 8, и вы получите десятичное число 10.

    🔟 Как считать до 10 в двоичной системе?

    Чтобы считать в двоичном формате, вы начинаете с 0, затем переходите к 1. Затем вы добавляете еще одну цифру, как при десятичном счете, когда переходите от 9 к 10. Вы добавляете еще одну цифру, так что теперь у вас есть две цифры. Итак, в двоичном формате вы переходите от 1 к 10, поскольку 1 — ваше последнее число.

    Итак, считая в двоичном формате, вы рассчитываете так:

    0
    1
    10
    11
    100
    101
    110
    111
    1000
    1001
    1010

    Вы можете найти десятичные числа от 0 до 100 (один 100) в таблице двоичных чисел на сайте ConvertBinary.com

    ✏️ Как преобразовать десятичное число в двоичное?

    Чтобы преобразовать десятичные числа в их двоичный эквивалент, у вас есть два варианта: вы можете либо использовать Decimal to Binary Converter на ConvertBinary.com, либо сделать это вручную.

    Если вы хотите научиться преобразовывать десятичное число в двоичное вручную, вы можете прочитать это руководство или просмотреть соответствующее руководство.

    🎓 Что обозначают двоичные числа?

    В математике и цифровой электронике двоичное число — это число, выраженное в системе счисления с основанием 2 или двоичной системе счисления, в которой используются только два символа: обычно «0» (ноль) и «1» (один). Система счисления с основанием 2 — это позиционная система счисления с основанием 2. Каждая цифра называется битом.

    Двоичный преобразователь в десятичный — w3resource


    Двоичное число:
    [Введите двоичное число, например 1110, в следующее поле и нажмите кнопку «Преобразовать». ]

    Десятичное число:

    Преобразование: двоичное в десятичное

    Двоичная система счисления:

    В математике и цифровой электронике двоичное число — это число, выраженное в двоичной системе счисления или системе счисления с основанием 2, которое представляет числовые значения с использованием двух разных символов: обычно 0 (ноль) и 1 (единица).Система с основанием 2 представляет собой позиционную систему счисления с основанием 2. Из-за ее простой реализации в цифровых электронных схемах с использованием логических вентилей двоичная система используется внутри почти всех современных компьютеров и компьютерных устройств. Каждая цифра называется битом.

    Десятичная система счисления:

    Десятичная система счисления (также называемая основанием десять) имеет основу десять, которая в десятичной системе счисления записывается как 10, как и база в каждой позиционной системе счисления.Это числовая база, наиболее широко используемая современными цивилизациями.

    Таблица преобразования двоичного числа в десятичное

    Двоичный
    Число
    Десятичное
    Число
    0 0
    1 1
    10 2
    11 3
    100 4
    101 5
    110 6
    111 7
    1000 8
    1001 9
    1010 10
    1011 11
    1100 12
    1101 13
    1110 14
    1111 15
    10000 16
    10001 17
    10010 18
    10011 19
    10100 20
    10101 21
    10110 22
    10111 23
    11000 24
    11001 25
    11010 26
    11011 27
    11100 28
    11101 29
    11110 30
    11111 31
    100000 32
    1000000 64
    10000000 128
    100000000 256

    Следующая: Преобразовать двоичное в шестнадцатеричное

    двоичных цифр

    Двоичная цифра может быть только 0 или 1

    В компьютерном мире « b inary dig it » часто сокращается до слова « bit »

    Более одной цифры

    Итак, есть только два способа получить двоичную цифру ( «0» и «1» или «Вкл. » И «Выкл.» )… а как насчет 2 или более двоичных цифр?

    Запишем их все, начиная с 1 цифры (можете сами проверить переключателями):

    Вот последний список боком:

    0000 0001 0010 0011 0100 0101 0110 0111 1000 1001 1010 1011 1100 1101 1110 1111

    И (без ведущих нулей) у нас есть первые 16 двоичных чисел:

    Двоичный: 0 1 10 11 100 101 110 111 1000 1001 1010 1011 1100 1101 1110 1111
    Десятичный: 0 1 2 3 4 5 6 7 8 9 10 11 12 13 14 15

    Это пригодится! Чтобы запомнить последовательность двоичных чисел, просто подумайте:

    • «0» и «1» {0,1}
    • , затем повторите «0» и «1» еще раз, но с «1» впереди: {0,1,10,11}
    • , затем повторите те с цифрой «1» впереди: {0,1,10,11,100,101,110,111}
    • и так далее!

    На каждом этапе мы повторяем все, что у нас есть до сих пор, но с 1 впереди.

    Теперь узнайте, как использовать двоичный код, чтобы считать на пальцах больше 1000:

    двоичных цифр … они удваиваются!

    Также обратите внимание, что каждый раз, когда мы добавляем еще одну двоичную цифру, мы удваиваем возможных значений.

    Почему двойной ? Потому что мы берем все предыдущие возможные значения и сопоставляем их с «0» и «1», как указано выше.

    • Итак, всего одна двоичная цифра имеет 2 возможных значения (0 и 1)
    • Две двоичные цифры имеют 4 возможных значения (0, 1, 10, 11)
    • Три имеют 8 возможных значений
    • Четыре имеют 16 возможных значений
    • Пять имеют 32 возможных значения
    • Шесть имеют 64 возможных значения
    • и др.

    Используя экспоненты, это может быть показано как:

    Число
    цифр
    Формула Настройки
    1 2 1 2
    2 2 2 4
    3 2 3 8
    4 2 4 16
    5 2 5 32
    6 2 6 64
    и т. Д… и др. .. и т.д …

    Пример: когда у нас есть 50 двоичных цифр (или 50 элементов, каждая из которых может иметь только две позиции), сколько разных способов это?

    Ответ: 2 50 = 2 × 2 × 2 × 2 × 2 … (пятьдесят из них)
    = 1 125 899 906 842 624

    Итак, двоичное число из 50 цифр может иметь 1 125 899 906 842 624 различных значения.

    Или, другими словами, он может отображать число до 1 125 899 906 842 623 (примечание: это на единицу меньше общего количества значений, потому что одно из значений равно 0).

    Пример: Начните месяц с 1 доллара и удваивайте его каждый день, через 30 дней вы станете

    миллиардером !

    2 30 = 2 × 2 × 2 × 2 … (их тридцать)
    = 1 073 741 824

    Шахматная доска

    Существует старая индийская легенда о короле, которого посетивший Мудрец вызвал на игру в шахматы. Король спросил: «Какой будет приз, если ты выиграешь?».

    Мудрец сказал, что ему просто нужно несколько зерен риса: одно на первом квадрате, 2 на втором, 4 на третьем и так далее, удваивая на каждом квадрате.Король был удивлен этой скромной просьбой.

    Итак, Мудрец выиграл, так сколько зерен риса он должен получить?

    На первом квадрате: 1 зерно, на втором квадрате: 2 зерна (всего 3) и так далее вот так:

    Квадрат Зерна Всего
    1 1 1
    2 2 3
    3 4 7
    4 8 15
    10 512 1 027
    20 524 288
    1 048 575
    30 53,6870,912
    1 073 741 823
    64 ???
    ???

    К 30 квадрату видно, что риса уже много! Миллиард зерен риса составляет около 25 тонн (1000 зерен — около 25 г. .. весила!)

    Обратите внимание, что сумма любого квадрата на 1 меньше, чем зерен на следующем квадрате (пример: сумма квадрата 3 равна 7, а квадрат 4 содержит 8 зерен). Таким образом, сумма всех квадратов представляет собой формулу: 2 n −1 , где n — номер квадрата. Например, для квадрата 3 сумма составляет 2 3 −1 = 8 1 = 7

    Итак, чтобы заполнить все 64 клетки на шахматной доске, потребуется:

    2 64 −1 = 18 446 744 073 709 551 615 зерен (460 миллиардов тонн риса),

    во много раз больше риса, чем во всем королевстве.

    Итак, к силе двоичного удвоения не стоит относиться легкомысленно (460 миллиардов тонн — это нелегко!)


    Зерна риса на каждом квадрате в экспоненциальном представлении
    Значения округлены, поэтому 53,6870,912 отображается как 5 × 10 8
    , что означает 5 с последующими 8 нулями

    (Между прочим, в легенде Мудрец раскрывается как Господь Кришна и говорит королю, что он не должен платить долг сразу, но может заплатить ему со временем, просто подавайте рис паломникам каждый день. пока долг не будет погашен.)

    Шестнадцатеричный

    Наконец, давайте посмотрим на особые отношения между двоичным и шестнадцатеричным.

    Есть 16 шестнадцатеричных цифр, и мы уже знаем, что 4 двоичных цифры имеют 16 возможных значений. Что ж, именно так они относятся друг к другу:

    Двоичный: 0 1 10 11 100 101 110 111 1000 1001 1010 1011 1100 1101 1110 1111
    Шестнадцатеричный: 0 1 2 3 4 5 6 7 8 9 А B С D E F

    Итак, когда люди используют компьютеры (которые предпочитают двоичные числа), намного проще использовать одну шестнадцатеричную цифру, чем 4 двоичные цифры.

    Например, двоичное число «100110110100» равно «9B4» в шестнадцатеричном формате. Я знаю, о чем предпочел бы написать!

    [PDF] = Глава 1. Двоичная система счисления. 1.1 Почему двоичный?

    1 Глава Двоичная система счисления. Почему двоичный? Система счисления, с которой вы знакомы и которую используете каждый день, …

    Глава 1 Двоичная система счисления

    1.1 Почему двоичная? Система счисления, с которой вы знакомы и которую используете каждый день, — это десятичная система счисления, также обычно называемая системой счисления по основанию 10.Когда вы выполняете вычисления, такие как 3 + 2 = 5 или 21-7 = 14, вы используете десятичную систему счисления. Эта система, которую вы, вероятно, усвоили в первом или втором классе, укоренилась в вашем подсознании; это естественный способ думать о числах. Конечно, дело не только в вас: все так думают — и всегда думали — о числах и арифметике. Существуют свидетельства того, что египтяне использовали десятичную систему счисления пять тысяч лет назад. Римская система счисления, преобладающая на протяжении сотен лет, также была десятичной системой счисления (хотя и организована иначе, чем арабская система счисления с основанием 10, с которой мы наиболее знакомы).Действительно, системы счисления с основанием 10, в той или иной форме, были наиболее широко используемыми системами счисления с тех пор, как цивилизация начала считать. Однако, имея дело с внутренним устройством компьютера, вам придется научиться думать в другой системе счисления, двоичной системе счисления, также называемой системой с основанием 2. Прежде чем рассматривать, почему мы могли бы захотеть использовать другую систему счисления, давайте сначала рассмотрим: почему мы используем основание 10? Ответ простой: у нас 10 пальцев. До того, как появились калькуляторы и компьютеры, мы считали по рукам (многие из нас до сих пор делают!).Представьте себе ребенка, считающего кучу пенсов. Он начинал: «Раз, два, три,…, восемь, девять». Достигнув девяти, следующая подсчитанная копейка составляет одну единую группу из десяти пенни. Затем он продолжает считать: «Одна группа по десять пенсов… две группы по десять пенсов… три группы по десять пенсов… восемь групп по десять пенсов… девять групп по десять пенсов…». По достижении девяти групп по десять пенсов плюс девять дополнительных пенсов, следующая посчитанная копейка составляет итого на данный момент: одна-единственная группа из ста пенни.Выполнив задание, ребенок может обнаружить, что у него осталось три группы по сто пенни, пять групп по десять пенни и два оставшихся пенни: 352 пенни. Более формально, система с основанием 10 — это позиционная система, где крайняя правая цифра — это позиция единиц (количество единиц), следующая цифра слева — позиция десятков (количество групп по 10), следующая цифра слева — позиция сотен (количество групп по 100) и так далее. Система счисления с основанием 10 состоит из 10 различных символов или цифр (0, 1, 2, 3,… 8, 9).В десятичной системе счисления мы записываем число в виде строки символов, где каждый символ является одной из этих десяти цифр, и для интерпретации десятичного числа мы умножаем каждую цифру на степень 10, связанную с позицией этой цифры. Например, рассмотрим десятичное число: 6349. Это число:

    6 3 4 9

    103 позиция (т. Е. Позиция тысяч)

    =

    6 103

    102 позиция (т. Е. Позиция сотен) 1

    3 102

    4 101

    9 100

    101 позиция (т.например, позиция десятков)

    100 позиций (то есть позиция единиц)

    Нет ничего существенно «проще» в использовании системы base-10. Это просто кажется более интуитивно понятным только потому, что это единственная система, которую вы широко использовали, и, опять же, тот факт, что она широко используется, объясняется тем, что у людей 10 пальцев. Если бы у людей было шесть пальцев, все мы использовали бы систему с основанием 6, и все мы нашли бы эту систему наиболее интуитивно понятной и естественной. Итак, давным-давно люди посмотрели на свои руки, увидели десять пальцев и решили использовать систему с основанием 10.Но сколько пальцев у компьютера? Учтите: компьютеры построены из транзисторов, и отдельный транзистор может быть только включен или выключен (два варианта). Точно так же устройства хранения данных могут быть оптическими или магнитными. Оптические запоминающие устройства хранят данные в определенном месте, контролируя, отражается ли свет от этого места или нет (два варианта). Точно так же магнитные запоминающие устройства хранят данные в определенном месте, намагничивая частицы в этом месте с определенной ориентацией.У нас может быть северный магнитный полюс, указывающий в одном направлении или в противоположном (два варианта). Компьютеры могут наиболее легко использовать два символа, и поэтому система с основанием 2 или двоичная система счисления является наиболее подходящей. Система счисления с основанием 10 состоит из 10 различных символов: 0, 1, 2, 3, 4, 5, 6, 7, 8 и 9. Система счисления с основанием 2 имеет ровно два символа: 0 и 1. Символы с основанием 10 называются цифрами. Символы с основанием 2 называются двоичными цифрами или для краткости битами. Все числа с основанием 10 состоят из цепочек цифр (например, 6349).Все двоичные числа состоят из цепочек битов (например, 1101). Так же, как мы сказали бы, что десятичное число 12890 состоит из пяти цифр, мы бы сказали, что двоичное число 11001 является пятибитным числом. Суть: все данные в компьютере представлены в двоичном формате. Фотографии вашего последнего отпуска хранятся на жестком диске — это все биты. Видео на YouTube, на котором кошка падает со стула, которое вы видели сегодня утром — кусочки. Ваша страница в Facebook — биты. Отправленный вами твит — биты. Электронное письмо от вашего профессора, в котором вам предлагается тратить меньше времени в отпуске, просматривать YouTube, обновлять страницу в Facebook и отправлять твиты — это тоже немного.Все биты. Чтобы понять, как работают компьютеры, вы должны говорить на этом языке. А язык компьютеров — это двоичная система счисления. 1.2. Двоичная система счисления. Рассмотрим снова пример ребенка, считающего стопку пенсов, но на этот раз в двоичном формате. Он начинал с первого пенни: «1». Следующий посчитанный пенни составляет одну единственную группу из двух пенни. Что это за номер? Когда ребенок с основанием 10 достигал девяти (высший символ в его схеме), следующий пенни давал ему «одну группу из десяти», обозначенную как 10, где «1» означало одну группу из десяти.Точно так же, когда ребенок с основанием 2 достигает единицы (наивысший символ в его схеме), следующий пенни дает ему «одну группу из двух», обозначенную как 10, где «1» указывает на одну группу из двух. Вернемся к дочернему элементу базы-2: следующий пенни составляет одну группу из двух пенни и одного дополнительного пенни: «11». Следующая добавленная копейка дает две группы по два, то есть одну группу из четырех: «100». «1» здесь указывает на совокупность двух групп по два, так же как «1» в числе 100 с основанием 10 указывает десять групп по десять. Выполнив задачу подсчета, ребенок с основанием 2 может обнаружить, что у него есть одна группа из четырех пенсов, ни одной группы из двух пенсов и один пенни в остатке: 101 пенни.Ребенок, считающий ту же стопку пенсов 2

    по основанию 10, сделает вывод, что было 5 пенсов. Итак, 5 в базе-10 эквивалентно 101 в базе-2. Чтобы избежать путаницы, когда используется основание, если оно не ясно из контекста, или при использовании нескольких оснований в одном выражении, мы добавляем нижний индекс к числу, чтобы указать основание, и пишем: 510

    1012

    Так же, как с десятичным обозначение, мы записываем двоичное число как строку символов, но теперь каждый символ — это 0 или 1.Чтобы интерпретировать двоичное число, мы умножаем каждую цифру на степень 2, связанную с позицией этой цифры. Например, рассмотрим двоичное число 1101. Это число:

    1 1 0 1

    23 позиция (т. Е. Позиция восьмерок)

    = 1 23 1 2 2

    22 позиция (т. Е. Позиция четверок)

    0 21 1 20

    21 позиция (т. Е. Позиция двоек)

    1310

    20 позиций (т. Е. Позиция единиц)

    Поскольку двоичные числа могут содержать только два символа 0 и 1, такие числа, как 25 и 1114000, не могут быть двоичными числа.Мы говорим, что все данные в компьютере хранятся в двоичном формате, то есть в виде единиц и нулей. Важно помнить, что значения 0 и 1 являются логическими значениями, а не значениями физической величины, например напряжения. Фактические физические двоичные значения, используемые для внутреннего хранения данных в компьютере, могут быть, например, 5 вольт и 0 вольт, или, возможно, 3,3 вольт и 0,3 вольт, или, возможно, отражение и отсутствие отражения. Два значения, которые используются для физического хранения данных, могут различаться в разных частях одного и того же компьютера.На самом деле имеет значение только то, что есть два разных символа, поэтому мы всегда будем называть их 0 и 1. Строка из восьми бит (например, 11000110) называется байтом. Набор из четырех битов (например, 1011) меньше байта и, следовательно, называется полубайтом. (Это вид юмора ботаников, которым славятся инженеры.) Идея описания чисел с помощью позиционной системы, как мы проиллюстрировали для оснований 10 и 2, может быть распространена на любую основу. Например, число 231 с основанием 4:

    2 3 1

    42 позиция (т. Е.е., шестнадцатая позиция)

    = 2 42

    41 позиция (т. е. позиция четверок) 3

    3 41 1 40

    4510

    40 позиций (т. е. позиция единиц)

    1.3 Преобразование двоичных чисел в десятичные Числа Мы, люди, относимся к числам, использующим десятичную систему счисления, тогда как компьютеры используют двоичную систему счисления. Нам нужно иметь возможность легко переключаться между представлениями двоичных и десятичных чисел. Преобразование двоичного числа в десятичное. Чтобы преобразовать двоичное число в десятичное, мы просто записываем двоичное число как сумму степеней двойки.Например, чтобы преобразовать двоичное число 1011 в десятичное, отметим, что крайняя правая позиция — это позиция единиц, а значение бита в этой позиции — 1. Итак, этот крайний правый бит имеет десятичное значение 1 20. Следующая позиция слева — это позиция двоек, и значение бита в этой позиции также равно 1. Итак, этот следующий бит имеет десятичное значение 1 21. Следующая позиция слева — это позиция четверок, и значение бита в этой позиции — 0. Крайняя левая позиция — позиция восьмерок, а значение бита в этой позиции — 1.Итак, этот крайний левый бит имеет десятичное значение 1 23. Таким образом: 10112

    1 23

    0 22

    1 21 1 20

    8 2 1

    1110

    Пример Выразите двоичное число 110110 как десятичное число. Решение:

    В качестве сокращенного средства преобразования двоичного числа в десятичное просто запишите значение позиции под каждым битом (т. Е. Запишите «1» под крайним правым битом, а затем «2» под следующим битом в слева, затем цифра «4» под следующим битом слева и т. д.), а затем сложите значения позиций для тех битов, которые имеют значение 1. Например, чтобы преобразовать двоичное число 10101 в десятичное, мы аннотируем значения позиций под значениями битов: 1 0 16 8

    1 4

    0 2

    1 1

    Затем мы складываем значения позиций для тех позиций, которые имеют битовое значение 1: 16 + 4 + 1 = 21. Таким образом, 101012

    2110

    Пример Выразите двоичное число 100000 как десятичное число. . Решение:

    4

    Теперь вы можете преобразовать двоичное число в эквивалентное десятичное.Теперь мы представим два разных метода преобразования в другом направлении: из десятичного в двоичный. Первый способ более интуитивен. Второй метод гораздо легче адаптировать для программирования на компьютере. Преобразование десятичного числа в двоичное число: метод 1 Первый метод преобразования десятичного числа в двоичное число влечет за собой выражение десятичного числа как суммы степеней 2. Чтобы преобразовать десятичное число x в двоичное: Шаг 1. Найдите наивысшая степень двойки меньше или равна x.Бинарное представление будет иметь единицу в этой позиции. Обозначим значение этой наивысшей степени двойки как y. Шаг 2. Теперь вычтите эту степень двойки (y) из десятичного числа (x), обозначив результат как z: z Шаг 3. Если z

    x y.

    0, готово. В противном случае, пусть x

    z и вернитесь к шагу 1 выше.

    Пример Преобразование десятичного числа 78 в двоичное. Решение. Подумайте про себя: «Я, какая наибольшая степень двойки меньше или равна 78?»

    Итак, наибольшая степень двойки, которая меньше 78, равна ____.Таким образом, двоичное представление 78 будет иметь

    ____________

    26

    64

    ___________

    25

    32

    ____________

    24

    ___________

    23

    16

    4

    _________

    21

    2

    ________

    20

    1

    Какая наибольшая степень двойки меньше или равна Ответ: ____________

    2

    6

    ___________

    2

    5

    32

    ____________

    2

    4

    ___________

    16

    2

    5

    4

    _________

    1

    2

    2

    ________

    20

    1

    Какая наибольшая степень двойки меньше или равна ___? Ответ: ____________

    26

    64

    ___________

    25

    32

    ____________

    24

    ___________

    23

    16

    Теперь, вычитая из 9

    , получаем 4

    __________

    22

    4

    _________

    21

    2

    ________

    20

    1

    2.Таким образом, теперь мы работаем с числом 2.

    Какая наибольшая степень двойки меньше или равна 2?

    ____________

    2

    6

    64

    ___________

    2

    5

    32

    ____________

    2

    4

    000

    000

    000

    000

    000

    000

    000

    __________

    2

    2

    4

    _________

    1

    2

    2

    ________

    20

    1

    Теперь, вычитая 2 из нашего числа 2, получаем 0, поэтомуЗаполнив нулями все оставшиеся позиции (то есть все позиции, у которых нет 1), мы получаем наш ответ: Десятичное число 78 в двоичном формате ____________

    2

    6

    64

    ___________

    2

    5

    32

    ____________

    2

    4

    ___________

    16

    2

    Пример Преобразование десятичного числа 201 в двоичное. Решение:

    6

    3

    8

    __________

    2

    2

    4

    _________

    1

    2

    2

    000

    ”Двоичные представления десятичных цифр от 0 до 15, показанные ниже.Десятичное число 0 1 2 3 4 5 6 7

    Двоичное число 0000 0001 0010 0011 0100 0101 0110 0111

    Десятичное число 8 9 10 11 12 13 14 15

    Двоичное число 1000 1001 1010 1011 1100 1101 1110 1111

    Вы можете интересно узнать о начальных нулях в таблице выше. Например, десятичное число 5 представлено в таблице как двоичное число 0101. Мы могли бы представить двоичный эквивалент 5 как 101, 00101, 0000000101 или с любым другим количеством ведущих нулей.Все ответы верны. Однако иногда вам будет указан размер хранилища. Когда вам задан размер места хранения, включите ведущие нули, чтобы показать все биты в месте хранения. Например, если вам предлагается представить десятичное число 5 как 8-битное двоичное число, ваш ответ должен быть 00000101. Преобразование десятичного числа в двоичное число: метод 2 Второй метод преобразования десятичного числа в двоичное число влечет за собой многократное деление десятичного числа на двоичное. десятичное число на 2, отслеживая остаток на каждом шаге.Чтобы преобразовать десятичное число x в двоичное: Шаг 1. Разделите x на 2, чтобы получить частное и остаток. Остаток будет 0 или 1. Шаг 2. Если частное равно нулю, все готово: переходите к шагу 3. В противном случае вернитесь к шагу 1, присвоив x значение самого последнего частного из шага 1. Шаг 3. Последовательность остатков образует двоичное представление числа. Пример. Преобразуйте десятичное число 53 в двоичное, используя второй метод, описанный выше. Решение:

    7

    Обратите внимание, что остаток от самого первого деления десятичного числа на два дает младший значащий бит (т.е.е., крайний правый бит) в двоичном представлении. Затем остаток от каждого деления представляет собой последовательные биты справа налево. Остаток от последнего деления (который всегда будет равен 1) предоставит самый старший бит (то есть крайний правый бит) в двоичном представлении. Пример Преобразование десятичного числа 201 в двоичное. Решение: Полный набор расчетов показан ниже.

    Вам может показаться, что второй метод проще в использовании, чем первый, но вы можете подумать: почему это работает? Объяснение того, почему работает этот алгоритм, представлено в Приложении 1-1.Итак, теперь вам должно быть удобно переключаться между двоичным и десятичным представлениями. Дано десятичное число, преобразуйте его в двоичное

    Десятичное число

    Двоичное число Преобразуйте двоичное число в десятичное

    1.4 Сложение двоичных чисел Мы знакомы с сложением десятичных чисел (с основанием 10). Но как складывать двоичные числа? Рассмотрим уравнение, с которым вы столкнулись в первом классе: 1 + 1 = 2 Если вы преобразовали каждое десятичное число в этом уравнении в двоичное, результат будет: 1 + 1 = 10 8

    Мы видим, что когда мы складываем два двоичных числа вместе, у нас есть перенос в следующий столбец (из столбца единиц в столбец двоек).Идея «переноса в следующий столбец» при сложении двоичных чисел аналогична переносу при сложении десятичных чисел. Подумайте о сложении десятичных чисел 8 и 4: 8 +4? Результат (который, как мы знаем, представляет собой десятичное число двенадцать) не может быть представлен одним десятичным символом (из которых возможны варианты 0, 1, 2, 3, 4, 5, 6, 8 и 9), содержащимся только в столбце единиц. . Мы должны перенести количество десять в столбец десятков (где оно становится 1, так как теперь оно находится в столбце десятков) и вычесть десять из двух чисел, которые мы складываем: 8 + 4 — 10 = 2.8 +4 12 Тот же принцип применяется к двоичному сложению (или сложению в любой системе счисления): если результат сложения является слишком большим числом, чтобы его можно было представить в виде одной цифры в системе счисления, мы переносим на следующую столбец и вычтите количество из суммируемых чисел. Подумайте о сложении двоичных чисел 1 и 1: 1 +1? Результат (который, как мы знаем, эквивалентен десятичному числу два) не может быть представлен одним двоичным символом (0 или 1), содержащимся только в столбце единиц.Мы должны перенести количество два в столбец двоек (где оно становится 1, поскольку оно находится в столбце двоек) и вычесть два из чисел, которые мы добавляем: 1 + 1-2 = 0. 1 +1 10 Подумайте. выполнения сложения трех двоичных чисел 1 + 1 + 1: 1 1 +1? Результат (который, как мы знаем, эквивалентен десятичному числу три) не может быть представлен одним двоичным символом (0 или 1), содержащимся только в столбце единиц. Мы должны перенести количество два в столбец двоек (где оно становится 1, поскольку теперь оно находится в столбце двоек) и вычесть два из трех чисел, которые мы складываем: 1 + 1 + 1 — 2 = 1.

    9

    1 1 +1 11 Сложение многобитовых двоичных чисел выполняется с использованием результатов, приведенных выше, по столбцам. То есть для каждого столбца используйте результаты: 0 + 0 = 0 1 + 0 = 1 1 + 1 = 0 с переносом 1 влево 1 + 1 + 1 = 1 с переносом 1 влево For Например, рассмотрим сложение двух двоичных чисел 11 и 01: 11 + 01 ?? Начиная с правого столбца, 1 + 1 дает 0, а 1 переносится влево: 1 11 + 01? 0 Теперь в левом столбце мы добавляем 1 + 1 + 0, что дает 0, с 1, перенесенный влево: 11 11 + 01 00 Наконец, мы рассматриваем крайний левый столбец (который состоит только из переходов из столбца вправо), в результате получаем: 11 + 01 100 Пример Выполните сложение двоичных чисел: +

    1 0 1 1 0 1 1 0 1 1 ————-

    +

    1 1 1 1 1 0 1 ———

    +

    1 0 1 1 0 1 1 0 1 1 ————-

    +

    1 1 1 1 1 0 1 ———

    Решение:

    10

    Пример Предположим, мы используем 4 бита для хранения целых чисел.Покажите сложение десятичных чисел 13 + 2 в двоичном формате. Решение:

    1.5. Переполнение. Ваш друг написал компьютерную программу, которая инициализирует натуральное число (т.е. неотрицательное целое) числом 4 294 967 293, а затем последовательно прибавляет 1 к числу, выводя результат на экран. Он считает, что в его программе должна быть ошибка, поскольку она дает результаты, показанные ниже. Он просит вашей помощи.

    Отложив в сторону очевидный вопрос: «Зачем ваш друг написал эту программу?», Вы решаете помочь.Вы замечаете, что программа, кажется, начинает хорошо, добавляя единицу и распечатывая правильный результат для первых двух итераций, но затем, по достижении значения 4 294 967 295, добавление единицы, кажется, «сбрасывает» значение обратно до нуля. Затем программа, кажется, возобновляет нормальную работу, добавляя единицу и распечатывая правильный результат. Что тут происходит? 11

    Чтобы изучить эту проблему, сначала рассмотрим компьютер, который использует 4 бита для хранения целых чисел. Число 15 будет сохранено как 1111.Каким будет результат вычисления 15 + 1 в двоичном формате? Выполнение двоичного сложения: +

    1 1 1 1 0 0 0 1 ——- 1 0 0 0 0

    Результат выглядит как 10000, что кажется правильным, поскольку это, в конце концов, эквивалентно до 16 в десятичной системе, и это правильный результат для 15 + 1. За исключением того, что это не тот ответ, который может дать компьютер! Правильный ответ на двоичное сложение 10000 требует пяти битов. Но компьютер (в нашем примере) хранит все целые числа в четырех битах.Все биты за крайними правыми четырьмя битами отбрасываются. Итак, ответ на 15 + 1 в двоичном формате на нашем 4-битном компьютере будет равен нулю (то есть 0000). Этот сценарий, когда для ответа требуется больше бит, чем доступно компьютеру, называется переполнением. Пример. Рассмотрим компьютер, который использует 4 бита для хранения целых чисел. Покажите двоичное вычисление 13 + 5. Решение:

    Поскольку сохраняются только крайние правые четыре бита, ответом будет 0010. Это, конечно, неверно, поскольку произошло переполнение.

    Переполнение памяти двоичных чисел — это реальная практическая проблема, которая была проклятием для многих программистов. Самая наглядная ошибка переполнения произошла в 1996 году, когда ракета Европейского космического агентства Ariane 5 взорвалась через 37 секунд после первого запуска, отправив 500 миллионов долларов в небытие. Программисты попросили бортовой компьютер сохранить 64-битную скорость в 16-битном пространстве. Остались только крайние правые 16 бит. Ракета попыталась отреагировать на ошибочную скорость, свернувшись на части.Итак, если компьютер хранит положительные целые числа в определенном количестве битов, какое наибольшее число может быть сохранено до того, как произойдет переполнение? Используя 3 бита, самое большое двоичное целое число, которое может быть сохранено: Используя 4 бита, самое большое двоичное целое число, которое может быть сохранено: Используя 5 битов, наибольшее двоичное целое число, которое может быть сохранено, составляет: Мы бы хотели заключить, что использование n битов Наибольшее двоичное целое число, которое может быть сохранено: 12

    Это на самом деле правильно, но приведенный выше аргумент не является доказательством, с которым вам должно быть комфортно.Чтобы определить наибольшее число, которое может быть сохранено в n битах, сначала рассмотрим наибольшее число, которое может быть сохранено в 4 битах:

    1 1 1 1

    23 позиция

    22 позиция

    21 позиция

    20 позиция

    Обратите внимание, что позиция самого старшего бита — это позиция 2 3. Если мы прибавим 1 к этому числу (и проигнорируем тот факт, что произойдет переполнение), результат будет 10000. Преобразовать это в десятичное число просто: в 24 позиции стоит только одна 1.Итак, одно число больше 1111 равно 24. Таким образом, 1111 должно быть равно 24 1. В более общем смысле, позиция самого старшего бита в n-битовом числе будет позицией 2n 1. Наибольшее число, которое может быть сохранено в n битах, будет строкой из n единиц (где крайний левый бит находится в позиции 2n 1). Теперь, если мы добавим 1 к этой строке из n единиц (и проигнорируем тот факт, что произойдет переполнение), результатом будет одна 1 в позиции 2n, за которой следуют нули. Таким образом, одно число, превышающее наибольшее число, которое может быть сохранено в n битах, равно 2n.Итак, наибольшее число, которое может быть сохранено в n битах, равно 2n 1. Пример Какое наибольшее положительное целое число может храниться на компьютере, если компьютер хранит положительные целые числа в четырех байтах? Решение:

    Теперь вы можете сообщить своему другу о проблеме в его программе? 1.5 Отрицательные двоичные числа Нам также необходимо представлять отрицательные целые числа. Как мы с этим справимся? У вас может возникнуть соблазн ответить: «Легко, просто используйте отрицательный знак!» Но это не сработает. Помните: все должно быть представлено в виде битов, то есть единиц и нулей! Отрицательный знак не является ни единицей, ни нулем.Наиболее интуитивным решением может быть представление знака и величины. При таком обозначении: пусть крайний левый бит представляет знак, скажем 0 для положительного и 1 для отрицательного. Остальные биты представляют величину.

    13

    Например, предположим, что мы храним целые числа в четырех битах со схемой знака и величины. Тогда 3 будет сохранено как 0011, а 2 будет сохранено как 1010. Как бы привлекательной ни казалась эта схема, она не используется. Чтобы понять, почему, рассмотрим сложение 3 и — 2.Используя сложение обычным способом, мы увидим 3 2

    0011 1010

    Но Итак, сложение непросто, если знаки двух аргументов различаются. Чтобы схема знак-величина работала, подумайте о том, что вам нужно сделать: вам нужно сначала определить, какой из двух аргументов имеет большую величину (игнорируя знаковый бит). Затем вы вычтите маленькую величину из большей величины. Затем вы присоедините к этому результату знак величины, имеющей более высокую величину. Хотя это можно было бы заставить работать, полученное оборудование было бы излишне сложным.Еще одна неприятность этой схемы заключается в том, что в ней есть два представления для нуля: (Математикам потребовалось более тысячи лет, прежде чем они смогли справиться с концепцией нуля … Конечно, мы должны сделать паузу, прежде чем случайно ввести систему с двумя нулями!) Все компьютеры представляют целые числа таким образом, чтобы можно было использовать как положительные, так и отрицательные числа. Чтобы представить положительные числа, просто используйте его двоичное представление, как мы делали раньше. Отрицательные двоичные числа представлены в так называемой «нотации с дополнением до двух».Чтобы найти двойное дополнение двоичного числа, замените каждую единицу на ноль и каждый ноль на единицу (т. Е. Инвертируйте все биты), а затем добавьте единицу к этой величине. Пример Каково дополнение двоичного числа 10010010 до двух? Решение:

    14

    Итак … как нам найти представление отрицательного десятичного числа? Сделайте следующее: сначала найдите двоичное представление числа без знака минус. Затем возьмите дополнение до двух. Результатом является представление отрицательного числа.Пример. Выразите десятичное целое число –13 как восьмибитовое двоичное число. Решение:

    Пример. Выразите десятичное целое число

    53 как восьмибитовое двоичное число.

    Решение:

    Пример Предположим, мы храним целые числа в 4 бита. Представьте 1 в двух дополнительных обозначениях. Решение:

    Существует удобный ярлык для поиска дополнения двоичного числа до двух. Начиная справа и двигаясь влево, оставьте все биты без изменений, пока не встретится первая 1.Оставив эту самую первую единицу без изменений, продолжайте движение влево, инвертируя все биты, начиная с этой точки. Например, мы отметили выше, что десятичное число +53 в двоичном формате равно 0 0 1 1 0 1 0 1 Чтобы найти дополнение до двух, мы начинаем справа и перемещаемся влево, останавливаясь на первой встреченной единице, которая в данном случае это крайний правый бит. Мы оставляем этот бит неизменным, но инвертируем все остальные биты влево, в результате получаем 1 1 0 0 1 0 1 1 15

    Пример Используйте метод быстрого доступа, чтобы определить два дополнительных представления -92 с использованием 8 бит Решение:

    Вы наверняка задаются вопросом: почему нотация дополнения до двух является хорошим представлением отрицательных чисел? Почему мы переворачиваем все биты и складываем единицу, а не, скажем, переворачиваем половину битов и складываем 5? Вот почему: двоичное число, добавленное к его двойному дополнению, дает ноль.Вот как это должно быть: число, добавленное к отрицательному, должно давать ноль. Например, предположим, что целые числа представлены 8 битами. В Разделе 1.3 вы определили, что двоичное представление числа 53 было 00110101. Два примера назад вы определили, что 53 было 11001011. Давайте прибавим 53 к 53: 53: -53:

    0 0 1 1 0 1 0 1 1 1 0 0 1 0 1 1 ————— 1 0 0 0 0 0 0 0 0

    Если мы используем только 8 бит, то сохраняются только восемь младших битов, и результат равен нулю (как и должно быть).Если мы представляем отрицательные числа с использованием записи дополнения до двух, то двоичное сложение с участием отрицательных чисел даст правильный результат. Сегодня каждый компьютер использует запись с дополнением до двух для хранения целых чисел. Пример. Предположим, что компьютер хранит целые числа в 4-х битах, используя запись с дополнением до двух. Покажите вычисление — 5 + 7. Решение:

    Вы можете согласиться с тем, что если мы представляем отрицательные числа с использованием записи дополнения до двух, то двоичное сложение с участием отрицательных чисел, кажется, дает правильные результаты, но вы можете подумать: почему это работает? Объяснение того, почему работает этот алгоритм, представлено в Приложении 1-2.16

    1.6 Дополнение до двух: более пристальный взгляд Предположим, у нас есть компьютер, который хранит целые числа в четырех битах. Если у нас есть только неотрицательные целые числа, то мы можем представить целые числа 0-15 с помощью четырех битов: Десятичное число 0 1 2 3 4 5 6 7 8 9 10 11 12 13 14 15

    Двоичное число 0000 0001 0010 0011 0100 0101 0110 0111 1000 1001 1010 1011 1100 1101 1110 1111

    Обратите внимание, что мы ограничены 16 возможными четырехбитными строками. Эти 16 уникальных строк показаны в правом столбце приведенной выше таблицы.Поскольку у нас есть 16 уникальных 4-битных строк, мы можем представить 16 уникальных десятичных чисел. Эти 16 десятичных чисел (0–15) показаны в левом столбце приведенной выше таблицы. Присвоение десятичного числа битовой строке выполняется обычным десятичным преобразованием в двоичное. Теперь предположим, что мы все еще хотим ограничиться хранением целых чисел в четырех битах, но мы хотим включать как положительные, так и отрицательные целые числа (используя нотацию дополнения до двух, поскольку мы знаем, что это работает!). Какие целые числа мы теперь можем хранить, если доступны только четыре бита? Должно быть ясно, что мы не можем хранить целые числа от –15 до +15 в нашем 4-битном пространстве.Диапазон от –15 до +15 включает 31 уникальное целое число, но поскольку у нас есть только 16 возможных битовых строк, мы можем сохранить только 16 уникальных целых чисел в пределах нашего четырехбитового распределения. Согласитесь, мы должны хранить как минимум целые числа от –7 до +7. Но этот диапазон будет состоять из 15 целых чисел, а у нас есть возможность хранить шестнадцать — так что есть место для еще одного! Должно быть +8? Или должно быть –8? Не может быть и того, и другого. Начнем с заполнения целых чисел 0-7: Десятичное число 0 1 2 3 4 5 6 7

    Двоичное число 0000 0001 0010 0011 0100 0101 0110 0111 1000 1001 1010 1011 1100 1101 1110 1111

    17

    Давайте заполним диапазон целых чисел от –1 до –7 по одному целому за раз.Начиная с –1, мы определяем, что это целое число представлено 1111 в дополнительном формате до 2, и у нас есть: Десятичное число 0 1 2 3 4 5 6 7

    -1

    Двоичное число 0000 0001 0010 0011 0100 0101 0110 0111 1000 1001 1010 1011 1100 1101 1110 1111

    Переходя к –2, мы определяем, что это целое число представлено числом 1110 в дополнительном виде: Десятичное число 0 1 2 3 4 5 6 7

    -2-1

    Двоичное число 0000 0001 0010 0011 0100 0101 0110 0111 1000 1001 1010 1011 1100 1101 1110 1111

    Добавляя два дополнительных представления для –3, –4, –5, –6 и –7, мы получаем: Десятичное число 0 1 2 3 4 5 6 7-7-6-5-4-3-2-1

    Двоичное число 0000 0001 0010 0011 0100 0101 0110 0111 1000 1001 1010 1011 1100 1101 1110 1111

    18

    У нас осталась одна битовая строка: 1000.Это должно быть +8 или –8? Было бы легко сделать так, чтобы эта строка, 1000, представляла +8. В конце концов, в двоичной системе +8 равно 1000. Но давайте рассмотрим –8. В дополнении до двух это целое число представлено… 1000! Итак, у нас действительно есть выбор: мы можем позволить нашей последней битовой строке 1000 быть либо +8, либо –8. Оба подойдут. Но один вариант намного лучше другого! Внимательно посмотрите на приведенную выше таблицу: для всех положительных чисел старший бит в четырехбитовой строке равен нулю. Для всех отрицательных чисел старший бит в четырехбитной строке равен единице.Если мы присвоим оставшуюся битовую строку (1000) целому числу –8, то по-прежнему будет иметь место тот случай, когда самый старший бит в четырехбитной строке будет равен единице для всех отрицательных чисел и нулю для всех положительных чисел. Это очень удобно — мы можем сразу определить, представляет ли число положительное или отрицательное целое число, просто проверив самый старший бит! Это путь! Окончательное присвоение: Десятичное число 0 1 2 3 4 5 6 7-8-7-6-5-4-3-2-1

    Двоичное число 0000 0001 0010 0011 0100 0101 0110 0111 1000 1001 1010 1011 1100 1101 1110 1111

    Подводя итог: если мы используем четыре бита для хранения целых чисел с использованием записи дополнения до двух, диапазон целых чисел, которые могут быть сохранены, составляет от — 8 до +7, и, учитывая четырехбитное двоичное число, мы можем сразу определить, положительный или отрицательный, если посмотреть на крайний левый бит.Если крайний левый бит равен 0, число положительное. Если крайний левый бит равен 1, число отрицательное. Как нам «обратить вспять» процесс их дополнения; то есть, учитывая, что 4-битное число с дополнением до двух хранится, скажем, как 1011, как я могу легко определить, что битовая строка хранит десятичное значение -5? Чтобы определить десятичный эквивалент числа, хранящегося в системе дополнения до двух, выполните следующие действия: Если крайний левый бит равен 0, число положительное. Просто преобразуйте битовую строку в десятичное число.Если крайний левый бит равен 1, число отрицательное. Чтобы определить десятичное число: o Возьмите два дополнения битовой строки (т. Е. Переверните все биты и добавьте один) o Преобразуйте битовую строку в десятичное число o Добавьте к результату отрицательный знак. 19

    Пример Предположим, мы сохраняем целые числа в 4-х битах, используя запись с дополнением до двух. Каково десятичное представление числа, хранящегося как 1111? Решение:

    Пример. Предположим, мы храним целые числа в 4-х битах, используя запись с дополнением до двух.Каково десятичное представление числа, хранящегося как 0101? Решение:

    Приведенные выше результаты предполагали, что мы сохраняли целые числа в нотации с дополнением до двух, используя 4 бита, но результаты можно легко обобщить для случая, когда мы сохраняем целые числа в нотации с дополнением до двух с использованием n бит. Пример Какое наибольшее положительное целое число может быть сохранено в n битах с использованием записи дополнения до двух? Решение:

    Пример Какое наименьшее отрицательное целое число может быть сохранено в n битах с использованием записи с дополнением до двух? Решение:

    Подведем итог важным результатам.Если мы сохраняем целые числа в нотации дополнения до двух с использованием n битов: самый левый бит работает как знаковый бит. Если крайний левый бит равен 0, число положительное; если крайний левый бит равен 1, число отрицательное. Диапазон значений, которые могут быть сохранены: Независимо от количества используемых бит, число ноль всегда представляется как

    20

    Независимо от количества используемых битов число 1 всегда представляется как Независимо от количества бит используется, самое положительное число: Независимо от количества используемых битов самое отрицательное число:

    Пример Каков десятичный эквивалент этого 32-битного дополнительного числа до двух? 1111 1111 1111 1111 1111 1111 1111 1100 Решение:

    Пример Покажите расчет — 5 + 3 в 4-битном представлении с дополнением до двух.Преобразуйте ответ обратно в десятичное число, чтобы убедиться, что ваш ответ правильный. Решение:

    Пример Покажите вычисление 6 + 3 в 4-битном представлении с дополнением до двух. Преобразуйте ответ обратно в десятичное число, чтобы убедиться, что ваш ответ правильный. Решение:

    21

    Пример. Показать расчет — 6 + (- 3) в 4-битном представлении с дополнением до двух. Преобразуйте ответ обратно в десятичное число, чтобы убедиться, что ваш ответ правильный. Решение:

    Переполнение распознается, когда мы складываем 2 числа одного знака и получаем результат с противоположным знаком.1.7 Двоичное вычитание В двоичном вычитании ничего нового не влечет! Чтобы вычислить a — b, мы вычисляем a + (- b), где — b представлено с использованием записи дополнения до двух. Пример. Покажите вычитание 3 2 на компьютере с дополнением до двух, который хранит целые числа в 4-х битах. Решение:

    Пример. Предположим, мы используем шесть битов для представления двоичного числа с дополнением до двух. Сделайте следующее вычитание. 011101 100101 Покажите приведенное выше вычитание со всеми числами, преобразованными в десятичные, и объясните свои результаты. Решение:

    22

    Мы имеем переполнение, если сложение двух положительных чисел дает отрицательное число или если сложение двух отрицательных чисел дает положительное число.Оказывается, есть быстрая проверка, которая может быть сделана, чтобы определить, произошло ли переполнение: если перенос в самый старший бит (который служит битом знака) отличается от переноса из самого старшего бита, произошло переполнение . 1.8 Шестнадцатеричная система счисления Нам часто приходится иметь дело с большими положительными двоичными числами. Например, предположим, что компьютеры подключаются к Интернету с помощью карты сетевого интерфейса (NIC). Каждой сетевой карте в мире назначается уникальный 48-битный идентификатор в качестве адреса Ethernet.Цель состоит в том, чтобы никакие две сетевые карты в мире не имели одинаковых адресов. Пример адреса Ethernet может быть: 000000000100011101011110011111111001001000110110. Другой пример: компьютерным инженерам часто приходится проверять содержимое определенного элемента в памяти компьютера. Например, вам может потребоваться посмотреть на переменную, которая хранится по адресу: 00000000000100101111111101111100. Вы, вероятно, согласитесь, что эти длинные двоичные строки было бы громоздко транскрибировать или считывать коллеге.Даже если вы полюбите двоичную систему счисления, вы все равно согласитесь с тем, что эти длинные строки — слишком хорошая вещь. К счастью, большие двоичные числа можно сделать намного более компактными — и, следовательно, с ними легче работать — если они представлены в так называемой шестнадцатеричной системе счисления с основанием 16. Вы можете спросить: двоичные числа также были бы более компактными, если бы они были представлены в базе 10 — почему бы просто не преобразовать их в десятичные числа? Ответ, как вы скоро увидите, заключается в том, что преобразование между двоичным и шестнадцатеричным форматом чрезвычайно просто — намного проще, чем преобразование между двоичным и десятичным.Шестнадцатеричная система счисления Шестнадцатеричная система счисления с основанием 16 состоит из 16 цифр (0, 1, 2, 3, 4, 5, 6, 7, 8, 9, A, B, C, D, E и F). Обратите внимание, что одиночный шестнадцатеричный символ A эквивалентен десятичному числу 10, одиночный символ B эквивалентен десятичному числу 11 и так далее, при этом символ F эквивалентен десятичному числу 15. Так же, как с десятичным или двоичным представлением. Мы снова записываем число в виде строки символов, но теперь каждый символ представляет собой одну из 16 возможных шестнадцатеричных цифр (от 0 до F).Чтобы интерпретировать шестнадцатеричное число, мы умножаем каждую цифру на степень 16, связанную с позицией этой цифры. Например, рассмотрим шестнадцатеричное число 1A9B. Указывая значения, связанные с позициями символов, это число показано как:

    1 A 9 B

    163 позиция

    162 позиция

    161 позиция

    23

    160 позиция

    Преобразование шестнадцатеричного числа в Десятичное число Чтобы преобразовать шестнадцатеричное число в десятичное, запишите шестнадцатеричное число как сумму степеней 16.Например, рассматривая шестнадцатеричное число 1A9B выше, мы преобразуем его в десятичное как: 1A9B = 1163

    A 162

    9 161

    B 160

    = 4096 + 10 (256) + 9 (16) + 11 ( 1) = 6811

    Пример Выразите шестнадцатеричное число 3CB как десятичное число. Решение:

    Преобразование десятичного числа в шестнадцатеричное.Самый простой способ преобразования десятичного числа в шестнадцатеричный — использовать тот же алгоритм деления, который вы использовали для преобразования десятичного числа в двоичное, но многократное деление на 16 вместо 2.Как и раньше, мы отслеживаем остатки, и последовательность остатков образует шестнадцатеричное представление. Например, чтобы преобразовать десятичное число 746 в шестнадцатеричное, мы действуем следующим образом: Остаток 16 | 746 | 46

    A

    | 2

    E

    | 0

    2 2EA

    Итак, десятичное число 746 = 2EA в шестнадцатеричном формате. Обратите внимание, что при использовании шестнадцатеричного представления, как и при использовании двоичного и десятичного представления, мы должны быть осторожны, чтобы понять основание. Когда мы говорим о числе «23», мы имеем в виду десятичное число 23 (в базе 10) или шестнадцатеричное число 23 (которое равно 35 в базе 10)? Если основание не ясно из контекста, его можно сделать явным, включив основание в качестве нижнего индекса, как в: 2316 3510.В некоторых текстах используется префикс 0x, чтобы указать, что число является шестнадцатеричным. То есть вместо 2316 в некоторых текстах будет использоваться запись 0x23. Возможно, вам будет приятно узнать, что на практике инженерам редко приходится переводить между десятичными и шестнадцатеричными числами (экзамены, проводимые профессорами-садистами, являются одним из таких редких случаев!). Преобразование шестнадцатеричного числа в двоичное. Инженерам часто приходится преобразовывать двоичное число в шестнадцатеричное, но, как мы и обещали, сделать это довольно просто.

    24

    Мы можем преобразовать непосредственно из шестнадцатеричной системы счисления в эквивалентное двоичное представление с помощью следующей процедуры: Преобразуйте каждую шестнадцатеричную цифру в четырехзначное двоичное число, независимо от других шестнадцатеричных цифр. Соедините полученные четырехбитные двоичные числа вместе. Например, чтобы преобразовать шестнадцатеричное число 4DA9 в двоичное, мы сначала преобразовываем каждую шестнадцатеричную цифру в четырехбитную строку: 4 = 0100

    D = 1101

    A = 1010

    9 = 1001

    , а затем объединяем результаты : Полученное двоичное число: 0100 1101 1010 1001.Мы можем отбросить ведущие нули (только из крайнего левого квартета!), Что даст нам: 4DA9 = 100110110101001 Пример Преобразование шестнадцатеричного числа 13F в двоичное. Решение:

    Преобразование двоичного числа в шестнадцатеричное Преобразование двоичного числа в шестнадцатеричное влечет за собой изменение процедуры преобразования шестнадцатеричного числа в двоичное. В частности, мы можем напрямую преобразовать двоичную нотацию в эквивалентное шестнадцатеричное представление, используя следующую процедуру: Начиная справа, собирайте биты группами по 4 биты. Преобразуйте каждую группу из 4 битов в эквивалентную шестнадцатеричную цифру. Соедините полученные шестнадцатеричные цифры. Например , чтобы преобразовать 110110101001 в шестнадцатеричное, мы собираем биты в группы по 4, начиная справа: 1101 1010 1001, а затем преобразуем каждый набор битов в шестнадцатеричную цифру: 1101 D

    1010 A

    1001 9

    Таким образом 110110101001 = DA9 Пример Преобразование двоичного числа 110101001 в шестнадцатеричное.Решение:

    25

    Пример Предположим, что первый байт переменной хранится в ячейке памяти с номером: 0 0 0 0 0 0 0 0 0 0 0 1 0 0 1 0 1 1 1 1 1 1 1 1 0 1 1 1 1 1 0 0 Мы говорим, что эта ячейка памяти является адресом переменной. Каков адрес этой переменной в шестнадцатеричной системе счисления? Решение:

    0000

    0000

    0001

    0010

    1111

    1111

    0111

    1100

    Опять же, шестнадцатеричная система счисления просто предоставляет нам более удобные средства передачи величин.Вместо того, чтобы говорить «Товар находится по адресу 1001000100111111», мы можем сказать: «Товар находится по адресу 913F». Пример Предположим, вам сообщили, что элемент находится по 32-битному адресу C356A20C. Какое значение имеет четвертый бит в этом адресе, если считать слева? Решение:

    Теперь вам может быть удобно переключаться между двоичным и шестнадцатеричным числами, но вы можете задаться вопросом: почему это так просто… почему я могу избежать наказания за преобразование каждой шестнадцатеричной цифры в двоичную, независимо от других цифр в номер? В конце концов, мы не можем просто посмотреть на десятичное число с основанием 10, такое как 732, и сразу же преобразовать его в основание 2, просматривая каждую цифру по отдельности.Хороший вопрос! Объяснение приведено в Приложении 1-3. Итак, теперь вам должно быть удобно переключаться между двоичным, десятичным и шестнадцатеричным представлениями.

    26

    1.9 Представление символов Теперь вы знаете, как положительные и отрицательные целые числа представляются в аппаратном обеспечении компьютера. Теперь мы обращаемся к представлению символов, таких как буквы алфавита, знаки препинания и другие разнообразные символы (например, $,%,? И т. Д.). Символы хранятся в компьютере с использованием кода Американского стандартного кода для обмена информацией — кода ASCII, показанного в таблице ниже.

    шестнадцатеричный символ ASCII 0 1 2 3 4 5 6 7 8 9 10 11 12 13 14 15

    0 1 2 3 4 5 6 7 8 9 ABCDEF

    NUL SOH STX ETX EOT ENQ ACK BEL BS TAB LF VT FF CR SO SI

    шестнадцатеричный символ ASCII 64 65 66 67 68 69 70 71 72 73 74 75 76 77 78 79

    40 41 42 43 44 45 46 47 48 49 4A 4B 4C 4D 4E 4F

    @ ABCDEFGHIJKLMNO

    шестнадцатеричный символ ASCII 16 17 18 19 20 21 22 23 24 25 26 27 28 29 30 31

    10 11 12 13 14 15 16 17 18 19 1A 1B 1C 1D 1E 1F

    ASCII Hex Symbol

    DLE DC1 DC2 DC3 DC4 NAK SYN ETB CAN EM SUB ESC FS GS RS US

    32 33 34 35 36 37 38 39 40 41 42 43 44 45 46 47

    шестнадцатеричный символ ASCII 80 81 82 83 84 85 86 87 88 89 90 91 92 93 94 95

    50 51 52 53 54 55 56 57 58 59 5A 5B 5C 5D 5E 5F

    20 (пробел) 21! 22 «23 # 24 $ 25% 26 и 27 ’28 (29) 2A * 2B + 2C, 2D 2E._

    96 97 98 9

    03

    100110 111111

    30

    1,12

    Преобразуйте следующее двоичное число в шестнадцатеричное: (a) (b) (c) (d)

    1.13

    Преобразуйте следующие шестнадцатеричные числа в двоичные: (a) (b) (c) (d)

    1.14

    ABCD 111111 101 23AC

    Сколько битов необходимо для представления: (a) (b) (c )

    1,15

    10110010101001001 10000000001 1111111 110011

    26 букв алфавита отдельные карты в колоде игральных карт лица на паре игральных костей

    Преобразуйте десятичное число 500 в BCD.

    31

    двоичное деление с десятичной запятой

    Двоичное деление против десятичного деления.Деление карандаша и бумаги, также известное как деление в столбик, является самым сложным из четырех арифметических алгоритмов. Мы просто удаляем кучу вещей, которые производят лишние ведущие нули.). Десятичная точность двоичных чисел с плавающей запятой. 0 ÷ 0 = бессмысленно. Подобно десятичной системе счисления, двоичное деление аналогично четырехступенчатому процессу: 1. Мы даже можем представлять числа с десятичной запятой, такие как 0,205 и 1,234, в двоичной системе. Деление на двоичное число осуществляется сдвигом точки на два шага 0 влево.Как и другие алгоритмы, он требует от вас решения более мелких подзадач того же типа. Однако он отлично работает с gcc -m32 test.c для создания 32-битного кода. Это всего лишь несколько шагов, и я могу решить это за вас; но позвольте мне сначала спросить вас: есть ли что-то в моем описании, которое вы не поняли? БОЛЬШОЕ СПАСИБО, РИК РЕГАН, Я ИСПОЛЬЗОВАЛ ВАШ АЛГОРИТМ ДЛЯ ОБУЧЕНИЯ БИНАРНОМУ ПОДРАЗДЕЛЕНИЮ СТУДЕНТОВ. Таким образом, вы можете преобразовать до 19 десятичных символов (макс. Найдите десятичное значение 111001 2: В вычислительных и электронных системах двоично-десятичный код (BCD) представляет собой класс двоичного кодирования десятичных чисел, где каждая цифра представлена фиксированное количество бит, обычно четыре или восемь.(Первый шаг долгого деления, как это обычно практикуется, объединяет несколько шагов и их подшагов в один. Возможно, вы можете отправить мне электронное письмо с подробностями (см. Мою страницу контактов) или продолжить обсуждение в одной из моих статей Дэвида Гэя. Спасибо за публикацию в этой серии статей (и отправив мне электронное письмо, чтобы сообщить, что она закончилась). Одна вещь, очень впечатляющая меня, — это презентационное изображение «Шаги десятичного деления». Да, потому что оно меньше или равно 831. (Помните, что эти являются двоичными числами, произносятся как «один-один», «один-ноль», «один-ноль-один» и т. д.10.01 (что на самом деле 25 * quadroot (5)). Процесс двоичного деления аналогичен процессу деления в столбик в десятичной системе счисления. Для двоичного числа с n цифрами: d n-1 … d 3 d 2 d 1 d 0. Определение двоичного представления с фиксированной точкой десятичного числа. В этом случае вместо деления на 2 мы умножаем числа после десятичной точки на 2 до тех пор, пока числа после десятичной точки не станут равными 0. Умножение 3. Простое преобразование между основами чисел, выполнение математических вычислений. Пусть a будет двоичным числом с именем делимое, которое будет разделено на другое (отличное от нуля) двоичное число b с именем делитель, чтобы получить частное q = a / b.Система счисления с основанием 2 также известна как двоичная система счисления. Мне нужен был один пример, который полностью показал бы деление в столбик. Я тоже использую свой двоичный калькулятор «обманом». Вместо того, чтобы угадывать, сколько раз наш делитель попадает в рабочий дивиденд (что может усложниться, если задействованные числа большие), при двоичном делении ответ будет… Я использовал его как модель для процедуры микроконтроллера в проекте электроники. [1] Двоичное деление: Метод двоичного деления также аналогичен методу, принятому в десятичной системе счисления.Вернемся к примеру введения, 1011.11 / 11. Превратите делитель (число, на которое вы делите) в целое число, сдвинув десятичную точку до упора вправо; одновременно переместите десятичную точку в делимом (число, которое вы делите) на такое же количество разрядов вправо. Теперь этот пример в десятичном формате для простоты, но наиболее распространенные форматы с плавающей запятой предназначены для двоичного. Пожалуйста, покажите работу. В сети есть много объяснений бинарного деления; один, который мне особенно нравится и который ближе всего подходит к тому, что я объяснил, — это Dr.Математическое «Длинное деление двоичного числа». Ах…. Я обнаружил, что в Ubuntu 64 v13.10 dtoa () от Дэвида М. Гея вызывал мертвый цикл, скомпилированный gcc test.c. Преобразование десятичного числа в двоичное с помощью метода деления Метод деления используется для преобразования только целой части десятичного числа в его эквивалент в двоичной системе счисления. Разделить 2. Дивиденд по-прежнему делится на делитель таким же образом, с единственной существенной разницей, заключающейся в использовании двоичного, а не десятичного вычитания. Как и в случае с десятичным делением, просто «добавьте 0», чтобы дивиденд был достаточно большим, чтобы его можно было разделить на: Большое спасибо за этот пост, это единственное, что мне пришлось отполировать перед экзаменом, теперь мне нужно идти и сдавать его, Пожалуйста, помогите мне с этим.Может ли кто-нибудь помочь мне с 111011/101? Это переносчики во время умножения (см. Мою статью о двоичном умножении). Я не понимаю вашего комментария о «мертвой петле». Примером двоичного числа является 001011. При оценке этого числа, например, 3,
    666666666667, мой двоичный преобразователь говорит, что оно равно 11,1110101010101010101010101010101010 при усечении до 36 знаков. ошибка или переполнение). Давайте расширимся до дробного двоичного представления с точкой (или запятой), разделяющей степени двойки с положительными и отрицательными показателями, так что например, g.Деление на двоичное число осуществляется сдвигом точки на один шаг влево. Преобразование из двоичного в десятичное включает в себя умножение значения каждой цифры (т.е. это означает, что с этого момента у нас будет двухзначный цикл (10). Недавно я прочитал несколько ваших статей. Я хотел подчеркнуть механическую процедуру, а не то, почему она работает (в десятичном или двоичном формате) Нет, потому что оно больше 8. В этом методе целая часть десятичного числа непрерывно делится до тех пор, пока мы не достигнем стадии, когда частное станет равным нулю.Повторяйте эти шаги, пока частное не станет равным 0. Завершающие нули будут сброшены, чтобы сформировать соответствующие подзадачи. Целая часть этого числа равна 10, а дробная часть — 0,16, и вместе они составляют число. Чтобы избежать беспорядка, я решил не отмечать заимствования, возникающие при вычитании. Напишите задачу как обычно: это очень полезно за благодарность от вас и вашего сайта, дорогой, большое спасибо за такое объяснение, но я не понял одной вещи: что, если у вас есть число, которое при делении начинается с нуля ….как этот Когда частное не является целым числом и процесс деления выходит за пределы десятичной точки, может произойти одно из двух: процесс может завершиться, что означает, что остаток равен 0; или Может быть достигнут остаток, который идентичен предыдущему остатку, который произошел после записи десятичных знаков. Для чего они? Он говорит, что 1011.11 / 11 равно 11.111010101010 (например, до 12 мест). Другой способ проверки — преобразовать операнды в десятичные числа, выполнить десятичное деление и затем преобразовать приблизительный десятичный ответ в двоичный.Вам не нужна докторская степень. преобразовать в числа с плавающей запятой. Как и другие алгоритмы, он требует от вас решения более мелких подзадач того же типа. Мне нужно полное объяснение того, как разделить 1111 на 11. Шаги по преобразованию двоичного числа в десятичное. Когда вы выполняете двоичное деление в столбик, вы можете обнаружить, что выполняете некоторые подшаги в своей голове в десятичном виде (например, 101-11 равно 5-3 = 2, что равно 10 в двоичном формате). Можете ли вы рассказать, какой инструмент используется для его производства? Услуга для всех нас «как работает математика?» люди! Делайте столбики с десятичными числами и просмотрите пошаговые инструкции по расчету.Итак, чтобы преобразовать десятичное число с плавающей запятой в двоичную форму, мы должны сначала преобразовать целую часть в двоичную форму. Три основных правила бинарного деления включают: 1. Делится ли 11 на 10? 11,75 / 3 = 3,916. Возьмите текущую сумму, сложите текущую цифру и разделите результат на 2. Однако в случае двоичных чисел операция проще, поскольку частное может иметь значение 1 или 0 в зависимости от делителя. Ошибка в функции Bigcomp в strtod () Дэвида Гэя, насколько просто «умножить» двоичное число на одну цифру, преобразовать приблизительный десятичный ответ в двоичный, десятичную точность двоичных чисел с плавающей запятой, правильное десятичное число в плавающее -Point с использованием больших целых чисел, 17 цифр доставят вас туда, как только вы нашли свой путь, интервал между двоичными числами с плавающей запятой, прямая генерация преобразований ошибок двойного округления в Kotlin, двойные ошибки округления в десятичных числах в двойные преобразования в плавающие, Максимальное количество десятичных цифр в двоичных числах с плавающей запятой.Но в отличие от других алгоритмов, здесь нет ограниченного набора «фактов», решающих все возможные подзадачи. Технически 88 превращается в 8 нулевых раз, поэтому мы должны записать 0, умножить 88 на 0, вычесть 0 из 8, а затем уменьшить 3. Исчисление: dx / dt = f (t) как dx = f (t ) * dt как x = интеграл f (t) dt, умножение вектора на действительное число, решение f (x) = 0 с помощью шага по времени x = x + f (x) * dt, шаг по времени: умный, тупой и средний Эйлер , Тригонометрические функции: cos (t) и sin (t). Вот пример: Алгоритм представляет собой серию шагов, каждый из которых имеет четыре подшага: переходят ли 88 в 8? Важное примечание: двоичное деление следует методу длинного деления, чтобы найти результат простым способом.11 входит в 101? Задать вопрос Задавали 3 года … Теперь я просто преобразовал двоичную строку в десятичный формат, выполнил операцию по модулю, чтобы узнать остаток. Два шага назад мы пытались разделить 400 на 88 раньше. Чтобы получить правильные 17 цифр, вам может потребоваться более 17 цифр. Очень хорошо написано и полезно. Пример. Вот почему я называю этот шаг 0. Разделите число на 2. Второе число умножается на первое и делится на первое; значения рассчитываются до 18 цифр как в десятичном, так и в двоичном формате.Давайте расширимся до дробного двоичного представления с точкой (или запятой), разделяющей степени двойки с положительными и отрицательными показателями, так что например, g. 110.11, таким образом, представляет; десятичная дробь. В моем примере есть многозначный делитель и есть ответ с остатком, который я написал как повторяющуюся десятичную дробь. Чтобы получить правильные 17 цифр, вам может потребоваться более 17 цифр. Результат преобразования десятичной дроби в двоичную в базовых числах. Чтобы преобразовать двоичную дробь в десятичную, начните справа с суммы 0. Решение этих подзадач деления требует оценки, предположений и проверки.Это означает, что с этого момента у нас есть двузначный цикл (45). Как мы увидим ниже, двоичные дроби демонстрируют некоторые интересные особенности поведения. В этом разделе мы начнем с рассмотрения того, как мы представляем дроби в двоичном формате. Чтобы использовать этот инструмент преобразования десятичных чисел в двоичные, вы должны ввести десятичное значение, например 308, в левое поле ниже, а затем нажать кнопку «Преобразовать». Обратите внимание, что хорошее понимание двоичного вычитания важно для проведения двоичного деления. значение